You are on page 1of 156

Published by

AMT PUBLISHiNG
Mathernafics Trust
University of Canberra ACT 2601
AUSTRALIA

Copyright C2001 AMT Publishing

Telephone: +61 2 6201 5137


AMTT Limited ACN 083 950 341
National Library of Australia Card Number and ISSN
Australian Mathematics Trust Enrichment Series 1SSN 1 326-0170
101 Problems in Algebra iSBN 1 876420 12 X

THE AUSTRALIAN MATHEMATICS TRUST

ENRiCHMENT SERIES
MiTT E EJ

Chairman
Editor

GRAHAM H POLLARD, Canberra AUSI'RALiA


PETER J TAYLOR, Canberra AUsTRALiA

WARREN J ATKiNs, Canberra AUSTRALIA

Eo J BARBEAU, Toronto CANADA


GEORGE BERZSENYI, Terra Haute USA
RON DUNKLEY, Waterloo CANADA

WALTER E MIENTKA, lincoln USA


N1KOLAY K0NsrANT1N0v, Moscow RussiA

ANDY Liii,

Edmonton

CANADA

JORDAN B TABOV, Sofia BULGARiA

JOHN WEBB, Cape Town SouFH AFRiCA

The books in this series are selected for their motivating, interesting
and stimulating sets of quality problems, with a lucid expository style
in their solutions. Typically, the problems have occurred in either
national or international contests at the secondary school level.

They are intended to be sufficiently detailed at an elementary level


for the mathematically inclined or interested to understand but, at
the same time, be interesting and sometimes challenging to the
undergraduate and the more advanced mathematician. lt is believed
that these mathematics competition problems are a positive
influence on the learning and enrichment of mathematics.

PREFACE
This book contains one hundred highly rated problems used in the training and testing of the USA international Mathematical Olympiad (IMO)
team. It is not a collection of one hundred very difficult, impenetrable
questions. Instead, the book gradually builds students' algebraic skills
and techniques. This work aims to broaden students' view of mathematics and better prepare them for possible participation in various mathematical competitions. It provides in-depth enrichment in important areas
of algebra by reorganizing and enhancing students' problem-solving tac-

tics and strategies. The book further stimulates students' interest for
future study of mathematics.

INTRODUCTION
In the United States of America, the selection process Leading to par-

ticipation in the International Mathematical Olympiad (IMO) consists


of a series of national contests called the American Mathematics Contest 10 (AMC 10), the American Mathematics Contest 12 (AMC 12),
the American Invitational Mathematics Examination(AIME), and the
United States of America Mathematical Olympiad (USAMO). Participation in the AIME and the USAMO is by invitation only, based on
performance in the preceding exams of the sequence. The Mathematical Olympiad Summer Program (MOSP) is a four-week. intense training of 24-30 very promising students who have risen to the top of the
American Mathematics Competitions. The six students representing the
United States of America in the IMO are selected on the basis of their
USAMO scores and further IMO-type testing that takes place during
MOSP. Throughout MOSP, full days of classes and extensive problem
sets give students thorough preparation in several important areas of
mathematics. These topics include combinatorial arguments and identities, generating functions, graph theory. recursive relations, telescoping
sums and products, probability, number theory, polynomials, theory of
equations. complex numbers in geometry, algorithmic proofs, combinatorial and advanced geometry, functional equations and classical inequalities.

Olympiad-style exams consist of several challenging essay problems. Cor-

rect solutions often require deep analysis and careful argument. Olympiad questions can seem impenetrable to the novice, yet most can he
solved with elementary high school mathematics techniques, cleverly applied.

Here is some advice for students who attempt the problems that follow.
Take your time! Very few contestants can solve all the given problems.

Try to make connections between problems. A very important


theme of this work is; all important techniques and ideas featured
in the book appear more than once!

Olympiad problems don't "crack" immediately. Be patient. Try


different approaches. Experiment with simple cases. In some cases,
working backward from the desired result is helpful.

Even if you can solve a problem, do read the solutions. They may
contain some ideas that did not occur in your solutions, and they

VIII

may discuss strategic and tactical approaches that can be used else-

where. The formal solutions are also models of elegant presentation that you should emulate, but they often obscure the torturous
process of investigation, false starts, inspiration and attention to
detail that led to them. When you read the solutions, try to reconstruct the thinking that went into them. Ask yourself. "What
were the key ideas?" 4How can I apply these ideas further?"

Go back to the original problem later, and see if you can solve it
in a different way. Many of the problems have multiple solutions,
but not all are outlined here.
All terms in boldface are defined in the Glossary. Use the glossary
and the reading list to further your mathematical education.

Meaningful problem solving takes practice. Don't get discouraged


if you have trouble at first. For additional practice, use the books
on the reading list.

ACKNOWLEDGEM ENTS
Thanks to Tiankai Liu who helped in proof reading and preparing solu-

Many problems are either inspired by or fixed from mathematical contests


in different countries and from the following journals:

High-School Mathematics, China


Revista Maternatic
Romania
Kvant, Russia
cite all the original sources of the problems in the solution part. We express our deepest appreciation to the original proposers
of the problems.
We did our best

ABBREVIATIONS AND NOTATIONS


Abbreviations
AHSME
AIME
AMC1O

AMC12

ARML
IMO
USAMO
MOSP

Putnam

St. Petersburg

American High School Mathematics


Examination
American Invitational Mathematics
Examination
American Mathematics Contest 10
American Mathematics Contest 12,
which replaces AIISME
American Regional Mathematics League
International J\Iathematical Olympiad
United States of America Mathematical Olympiad
Mathematical Olympiad Summer Program
The William Lowell Putnam Mathematical
Competition
St. Petersburg (Leningrad) Mathematical
Olympiad

Notations for Numerical Sets and Fields


Z
N
N0

Q
Q+
Q

R
IR+

R
R'2

the set of integers


the set of integers modulo n
the set of positive integers
the set of nonnegative integers
the set of rational numbers
the set of positive rational numbers
the set of nonnegative rational numbers
the set of n-tuples of rational numbers
the set of real numbers
the set of positive real numbers
the set of nonnegative real numbers
the set of n-tuples of real numbers
the set of complex numbers

PREFACE

vii

INTRODUCTION

ix

ACKNOWLEDGEMENTS

xi

ABBREViATiONS AND NOTATiONS


1. INTRODUCTORY PROBLEMS
2. ADVANCED PROBLEMS

xiii
1

13

3. SOLUTIONS TO INTRODUCTORY PROBLEMS 27

4. SOLUTIONS TO ADVANCED PROBLEMS

65

GLOSSARY

131

FURTHER READING

137

iNTRODUCTORY PROBLEMS

INTRODUCTORY PROBLEMS

problem

Let a. b. and c be real and positive parameters. Solve the equation

ax =

CX-r

Problem 2
Find the genera! terni of the sequence

=
for

bi +

ax +

by

3.

- cx.

x1 =

alln E N.

Problem 3
x,, be a sequence of integers such that

Let x1. x2

n:

(i)

(ii) x1 + x2 +

= 19:

=99.

(iii)

Determine the minimum aiid niaxiniuni possible values of

Problem 4
The function f. defined by
1(x)
where a.

b.

c. and

= c.r+ d

d are nonzero real nunibers. has the j)rOperties

f(19) =
for

ax +

19.

all values of x. eXCCl)t


Find
range of f,

f(97) =
--

97.

and

f(f(x)) =

and

1. Introductory Problems

Problem 5
Prove that

(ab)2 <
8a

for all a

a+b

(ab)2

8b

b> 0.

Problem 6
Several (at least two) nonzero numbers are written on a board. One may

erase any two numbers. say a and b, and then write the numbers a +
and b

instead.

Prove that the set of numbers on the board, after any number of the
preceding operations, cannot coincide with the initial set.

Problem 7
The polynomial

may be written in the form


ao + a1y + a2y2

where y = x + 1 and
Find a2.

+ a15y16 + a17y17,

are constants.

Problem 8
Let a, b, and c be distinct nonzero real numbers such that

a+-=b+-c
1

Prove that

1.

Problem 9
Find polynomials 1(x), g(x), and h(x), if they exist, such that for all x,
(

3x+2
2x+2

ifx<1
ifi

ifx>0.

i. Introductory Problems
Problem 10
Find all real numbers x for which

8x+27x

12x + 18x

Problem 11
Find the least positive integer m such that

(2n'\
I

for all positive integers n.

Problem 12
Let a, b, c, d, and e be positive integers such that

Find the maximum possible value of max{a, b, c, d, e}.

Problem 13
Evaluate
3

2001

1!+2!+3!

1999!+2000!+2001!

Problem 14

Letx=

1, a ER.

Find all possible values of x.

Problem 15
Find all real numbers x for which

+ lix + 12T =

13X

+ 14x.

1. Introductory Problems

Problem 16

Let f N x N

be a function such that f(1, 1) =

2,

f(rn + 1. n) = f(rn. n) + rn and f(rn, n + 1) = f(n2, n)

for all rn, n E N.

Find all pairs (pq) such that f(p,q) = 20th.

Problem 17
Let f be a function defined on [0, 1] such that

f(0) =
for all a

1(1)

and (f(a) f(b)I < a bI.

in the interval [0, 1].

Prove that
1

1(a) f(b)( <

Problem 18
Find all pairs of integers (x, y) such that
x3 + y3 = (x + y)2.

Problem 19
2

Let f(x) =
for real numbers x.
4x + 2
Evaluate
1

/2000

Problem 20
Prove that for ii

6 the equation
1

x1

x2

has integer solutions.

Problem 21
Find all pairs of integers (a, b)
is divisible by x2 x

1.

such

that the polynomial ax'7 + bx16 + I

1. IntroductorY Problems

Problem 22
Given a positive integer n, let p(n) be the product of the non-zero digits

of n. (If n has only one digit, then p(n) is equal to that digit.) Let
+p(999).
What is the largest prime factor of S?

Problem 23
Let

be a sequence of nonzero real numbers such that


xn

n = 3,4
Establish necessary and sufficient conditions on x1 and x2 for
for

to be

an integer for infinitely many values of ri.

Problem 24
Solve the equation

3x

Problem 25
For any sequence of real numbers A = {ai,a2,as,.
the sequence {a2 a1, a3 a2, a4 a3..

of the sequence
Find a1.

}, define

to be

}. Suppose that all of the terms

are 1, and that a1g =

a92

0.

Problem 26
Find all real numbers x satisfying the equation

2x+3x

4X+6X9X =1.

Problem 27
Prove that
16

<17.

Problem 28
Determine the number of ordered pairs of integers (rn n) for which mn

m3 + n3 + 99mn =

1. Introductory Problems

Problem 29
Let a, b, and c be positive real numbers such that a + b + c

4 and

ab 4-bc+ca 4.
Prove that at least two of the inequalities
abl

2,

lbcl 2,

Ical

are true.

Problem 30
Evaluate
1

Problem 31
Let 0 < a < 1. Solve

ax

for positive numbers x.

Problem 32
What is the coefficient of x2 when

is expanded?

Problem 33
n be distinct positive integers.
Find the maximum value of lxm
where x is a real number in the
interval (0,1).
Let

Problem 34
Prove that the polynomial

are distinct integers, cannot be written as the product of two non-constant polynomials with integer coefficients, i.e., it is
where ai. a2, ..
irreducible.

Problem 35
Find all ordered pairs of real numbers (x. y) for which:

(1+x)(1+x2)(1+x4)
and

(1+y)(1+y2)(1+y4) =

1+x7.

Problem 36

Solve the equation


2(2x

1)x2 + (2z2 2)x =

for real numbers x.

Problem 37
Let a be an irrational number and let n be

an integer greater than 1.

Prove that
is an irrational number.

Problem 38
Solve the system of equations

= x2(x4+x5x2)

(x1 x2 -I-- x3)2

= x3(x5+x1x3)
=
(x4 x5 + x1)2 = x5(x2+x3x5)
(x5 xi +
= x1(x3+x4x1)
(x2 x3 + x4)2

(x3 x4 + x5)2

for real numbers Xl, x2,

x4, x5.

Problem 39
Let x, y, and z be complex numbers such that
x

+z=

2,

+ z2

and

STAATS.UL.NIV)

11

4.

Xy+z1 + yz+x1 + zx+y1

1. Introductory Problems

Problem 40
Mr. Fat is going to pick three non-zero real numbers and Mr. Taf is going
to arrange the three numbers as the coefficients of a quadratic equation
_x2 +_x+_=O.

Fat wins the game if and


distinct rational solutions.
Who has a winning strategy?
Mr.

only if the resulting equation

has two

Problem 41
Given that the real numbers a, b, c, d, and e satisfy simultaneously the
relations

a+b+c+d+e=8anda2-j-b2+c2+d2+c2 =16,
determine the maximum and the minimum value of a.

Problem 42
Find the real zeros of the polynomial
Pa(S) = (x2 + 1)(x

1)2 ax2,

where a is a given real number.

Problem 43
Prove that
1

2n1

2n

for all positive integers n.

Problem 44
Let

P(x) =
a nonzero polynomial with integer coefficients such that P(r) =
P(s) = 0 for some integers r and s. with 0 < r < s.
Prove that ak < s for some k.

Problem 45
Let rn be a given real number.
Find all complex numbers x such that

(S

(X\
/

+(

2
=m +772.

Problem 46
The sequence given by xo =

a,

.r1 =

b,

and

11

is periodic.

Prove that ab =

1.

Problem 47
Let a, b, c, and d be real numbers such that
(a2 +b2 i)(c2 +d2 1)>

(ac+bd

1)2.

Prove that
a2

+ b2> 1

and c2

+ d2> 1.

Problem 48
Find

all complex numbers z such that


(3z + 1)(4z + 1)(6z + 1)(12z + 1) =

2.

Problem 49
be the zeros different from 1 of the polynomial

Let

P(x) =

1,

n 2.

Prove that
1

ix1

ix2

+...+

ni

Problem 50

Let a and b be given real numbers. Solve the system of equations

V1_x2+y2

for real numbers x and y.

ADVANCED PROBLEMS

Z ADVANCED PROBLEMS
Problem 51
Evaluate
(2000'\

(2000\

(2000\

(2000

Problem 52
Let x, y, z be positive real numbers such that x4 + y4 -i- z4 =
Determine with proof the minimum value of
x3

1x 8+

ly8

z3

1z8

Problem 53
Find all real solutions to the equation
2X

+ 3X + 6X = x2.

Problem 54
Let {an}ni be a sequence such that

and

for all n E N.
Find an explicit formula for

Problem 55
Let x, y, and z be positive real numbers. Prove that
2;

z+

y)

1.

1.

2. Advanced Problems

14

Problem 56
Find, with proof, all nonzero polynomials f(z) such that

f(z2) + f(z)f(z +1) =0.


Problem 57

Let f N

be a function such that f(n + 1) > f(n) and

f(f(n)) =

3n

for all n.
Evaluate 1(2001).

Problem 58
Let F be the set of all polynomials f(x) with integers coefficients sueh
that f(x) = 1 has at least one integer root.
For each integer k > 1. find
the least integer greater than 1 for
which there exists f E P such that the equation f(x) = rnk has exactly
k distinct integer roots.

Problem 59
Let x1 =

and
=

1,

i.
Prove that

for ii

Problem 60
Suppose that f

22

.r1

x2

22

is a decreasing function such that for all

x,y E

f(x + y) + f(f(x) + 1(y)) = 1(1(1 + f(y)) + f(y + 1(x))).


Prove

that f(f(x)) =

x.

15

Problem 61
Find all functions f : Q

Q such that

f(x + y) + f(x
for

y)

= 2f(x) + 2f(y)

all x,y EQ.

Problem 62
Let <a < 1.
Prove that the equation
x3(x + 1) = (x + a)(2x + a)
has

four distinct real solutions and find these solutions in explicit form.

Problem 63
Let a, b, and c be positive real numbers such that abc =
Prove that
1
1
I
<1.
+

1.

a+b+1

c+a+1

b+c-I-1

Problem 64
Find all functions f, defined on the set of ordered pairs of positive integers, satisfying the following properties:

f(x, x) = x, f(x, y) = f(y, x). (x + y)f(x. y)

yf(x. x + y).

Problem 65
Consider n complex numbers zk. such that IzkI
2
1, Ic =
Prove that there exist c1. a2....
E fi. 1} such that, for any vi
e1z1

Problem 66
Find a triple of rational numbers (a.

<2.

c) such that

n.
fl,

2. Advanced

16

Problem 67
Find the minimum of
+ log12

(X2-_

log11

where

x1,x2,. ..

are real numbers in the interval

1).

Problem 68
Determine x2 +

+ z2 + w2 if

22_12+ 22_32+22_52

=1,

+4252

=1,

42_12 + 42_32

62_12

+6232+6252+6272 =L

82_12 + 82_32

-L

82_52 + 82_72 =1

Problem 69
Find all functions f R
:

such that

f(xf(x) + f(y)) = (f(x))2 +


for all x,y E

Problem 70
The numbers 1000, iOOl,
,2999 have been written on a board.
Each time, one is allowed to erase two numbers, say, a and b, and replace

them by the number

min(a. b).

After 1999 such operations, one obtains exactly one number c on the
board. Prove that c < 1.

Problem 71
Let a1,a2,. .
be real numbers, not all zero.
Prove that the equation
.

has at most one nonzero real root.

2.

Advanced Problems

Problem '72
real numbers defined by a1 = t and
Let {an) be the sequence of

=
for n

1.

For bow many distinct values

oft

do we have a1998

0?

Problem '73
R

(a) Do there exist functions f :

f(g(x)) = x2

and

and g R

such that

g(f(x)) =

for all x E R?

and 9:

(b) Do there exist functions f: R

f(g(x)) = x2

and

g(f(x))

such that

for all x E R?

Problem 74

LetO<a1 a2

<

O<b1 <b2

Suppose that there exists 1


b2

for

i >

k n such

berealnumberssuchthat

for 1

that

k and

k.

Prove that
a1a2

b1b2

Problem 75
Given

eight non-zero real numbers a1, a2,.


a8. prove that at least one
of the following six numbers: a1a3 + a2a4, a1a5 + a2a6, a1a7 + a2a8,
a3a5 + a4a6, a3a7 + a4a8.
+ a6a8 is non-negative.

Problem '76
Let a, b and
Prove that

be

positive real numbers such that abc

ab

bc

ca

1.

a5+b5+ab+b5+cS+bc+c5+a5+caL

2. Advanced Problems

18

Problem 77
Find all functions f: R * R such that the equality

f(f(x) + y) = f(x2

y)

+ 4f(x)y

holds for all pairs of real numbers (x. y).

Problem 78
Solve the system of equations:
3x y
x2

=3

y2

x+3y

11

=0

Problem 79
Mr. Fat and Mr. Taf play a game with a polynomial of degree at least 4:

+... +_x + 1.
They fill in real numbers to empty spaces in turn. If the resulting polynomial has no real root, Mr. Fat wins; otherwise, Mr. Taf wins.
If Mr. Fat goes first, who has a winning strategy?

Problem 80
Find all positive integers k for which the following statement is true:
F(x) is a polynomial with integer coefficients satisfying the condition
for

then F(0)= F(1)=

c=0.1

k+1,

=F(k+ 1).

Problem 81
The Fibonacci sequence

is given by

(nEN).
Prove that
2n+2 '

for all n 2.

2n2

bf2n

if

19

ced ProblemS

Problem 82
R for which there exists a strictly monotonic
Find all functions u :
function f : R p R such that

f(x + y) = f(x)u(y) + f(y)


for all x, y c R.

Problem 83
Zn be

Let

complex numbers such that

IzlI+Iz2H+IzflI=1.
Prove

that there exists a subset S of {z1, Z2,..

such that

Problem 84
A polynomial P(x) of degree n 5 with integer coefficients and n distinct
integer roots is given.

Find all integer roots of P(P(x)) given that 0 is a root of P(x).

Problem 85
Two real sequences x1, x2,

and Yi

Y2.

are defined in the following

way:

and
Yn+1

for all n 1. Prove that 2 <

yn

+ yi +
<3 for all a > 1.

Problem 86
For a polynomial P(x), define the difference of P(x) on the interval [a. b]
([a,b), (a,b). (a.b]) as P(b) P(a).
Prove that it is possible to dissect the interval [0. 1[ into a finite number
of intervals and color them red and blue alternately such that. for every
polynomial P(x). the total difference of P(.r) on red intervals
is equal to that of P(x) on blue intervals.
What about cubic polynomials?

2. Advanced Problems

20

Problem 87
Given a cubic equation

x3 + _x2 + _x + = 0,
Mr. Fat and Mr. Taf are playing the following game. In one move, Mr.
Fat chooses a real number and Mr. Taf puts it in one of the empty spaces.
After three moves the game is over. Mr. Fat wins the game if the final
equation has three distinct integer roots.
Who has a winning strategy?

Problem 88
Let n > 2 be an integer and let f R2
any regular n-gon A1A2 .

be a function such that for

. .

f(A1) + f(A2) + .

f is the zero function.


Problem 89
Let p be a prime number and let f(x) be a polynomial of degree d with
integer coefficients such that:

(i) f(0) =

0,

f(l) =

1;

(ii) for every positive integer n, the remainder upon division of


by p is either 0 or 1.
Prove that d p

1.

Problem 90
Let n be a given positive integer.

with a0 =

Consider the sequence a0,a1,

ak =

4
TI

fork=l,2..n,
Prove that
1

_!
TI

<

1.

and

2.

21

Advanced problems

Problem 91

Let ai,a2,."

be nonnegative real numbers. not all zero.

(a) Prove that


positive real root R.

(b) Let A =

a2 and

Prove that AA

= 0 has precisely one

ja3.

B=

<RB.

Problem 92
Prove that there exists a polynomial P(x, y) with real coefficients such
that P(x, y) 0 for all real numbers .r and y. which cannot be written
as the sum of squares of polynomials with real coefficients.

Problem 93
For each positive integer Ti. show that there exists a positive integer k
such that
/c = f(x)(x + 1)2n +
+ 1)
for some polynomials f.g with integer coefficients, and find the smallest
such Ic as a function of n.

Problem 94
Let x be a positive real number.

(a) Prove that


(x

(ii 1)!
1
1).. (x + n) I
.

(b) Prove that

(ni)!

2. Advanced

22

Problem 95
Let ii 3 be an integer, and let

Xcs={1.2

n3}

be a set of 3n2 elements.


Prove that one can find nine distinct numbers
such that the system

c7,

a2x+h2y+c2z =

a3x+b3y+c3z

(1 =

1.

2, 3) in X

has a solution (XO, yo, zO) in nonzero integers.

Problem 96
he positive real numbers.

Let n 3 be an integer and let Xi. i2.


Suppose that

=
1

Prove

that

Problem 97
Let x1,x2

be

distinct real numbers. Define the polynomials

P(x) =

(x x1)(.r

(1

and

Q(x)=P(x)(

\.iX1

..

XX2

-+.

Let y1,y2.. . . 'Ynl be the roots of Q. Show

tJ

<

that

2.

Advanced Problems

Problem 98
Show that for any positive integer fl. the polynomial

f(x) =

(x2

+1

cannot be written as the product of two non-constant polynomials with


integer coefficients

Problem 99
be functions such that

Let 11,12.13 R

a1f1 + a2]'2 + a3f3

monotonic for all a1,a2,a3


Prove that there exist c1, c2. C3
is

R.

R. not all zero. such that

cifi(x) +c2f2(x)
for all x

+c3f3(x) = 0

Ift.

Problem 100
be the sums of
be variables, and let
Let x1.x2,
nonempty subsets of
elementary symmetric polynomial in
be the
Let pk(xl
the yj (the sum of every product of A- distinct yjs).
For which k and n is every coefficient of Pk (as a polynomial in x1,. ..
. .

even?

For example, if n =

2.

Pi =
P2 =
P3 =

+ s2 and

then yi' Y2. y3 are Xl.

+ Y2 +
Y1Y2

+ 2x2,

+ 112Y3 + Y3Y1 =

Y1Y2Y3

X1X2

Problem ioi.
Prove that there exist 10 distinct real numbers al,a2..
the equation

(xai)(x_a2)...(xaio)
has exactly 5 different real roots.

= (X+al)(X+a2)

aio

such that

(x+aio)

SOLUTiONS TO
iNTRODUCTORY PROBLEMS

3. SOLUTIONS TO
PROBLEMS
problem 1 [Romania 1974]
Let a, b, and c be real and positive parameters.
Solve the equation

\/c+ax= v'bax -f-s/c


Solution 1
It is easy to see that x =

0 is a solution. Since the right hand side is a


decreasing function of x and the left hand side is an increasing function
of x, there is at most one solution.
Thus x = 0 is the only solution to the equation.

Problem 2
Find the general term of the sequence defined by x0 =

_2

3,

and

for all n C N.

Solution 2
We shall prove by induction that

+ 3. The claim is evident for

n=O,1.

Fork 1, if Xk_1 =k+2andxk=k+3, then


kxk =(k+2)2 k(k+3)=k+4,
as desired.

This completes the induction.

3. Solutions to Introductory Problems

28

Problem 3 [AHSME 1999]


Let x1. x2,
, x7, be a sequence of integers such that
.

n;

(i)

(ii)

19;

X1

(iii)

Determine the minimum and maximum possible values of

Solution 3
Let a, b, and c denote the number of is, is, and 2s in the sequence,
respectively. We need not consider the zeros. Then a, b, c are nonnegative
integers satisfying

a+b+2c= l9anda+b+4c=99.
It follows that a = 40c and b

593c, where 0

19 (since b 0),

so

19+6c.
(a = 40,b = 59), the lower bound (19) is achieved.
When c = 19 (a = 21, b = 2), the upper bound (133) is achieved.
When c =

Problem 4 [AIME 1997]


The function f, defined by

f(x)

ax + b

cx+d'

where a, b, c, and d are nonzero real numbers, has the properties

1(19) =

19,

f(97) =

97,

and

for all values of x, except

Find the range of f.

Solution 4, Alternative 1
For all x, f(f(x)) = .r, i.e.,

(ax + b'\

a(

\cx+d)
(ax+b\
cx + dj

f(f(x)) =

to Introductory Problems

i.e.

29

(a2 + bc)x + b(a + d)

c(a+d)x+bc+d2
i.e.

X.

c(a + d)x2 + (d2 a2)x b(a + d) =

0,

implies that c(a + d) = 0. Since c 0, we must have a = d.


The conditions f(19) = 19 and f(97) = 97 lead to the equations
which

and
Hence

(972

It follows that a =

58c,

192)c

972c=297a+b.

= 2(97

19)a.

which in turn leads to b =

1843
f(x)= 58x.x58

1843c.

Therefore

1521

which never has the value 58.

Thus the range of f is R {58}.

Solution 4, Alternative 2

The statement implies that f is its own inverse. The inverse may be
found by solving the equation
ay + b
cy + d

fory. Thisyields
dxb
cx +

The nonzero numbers a. b, c, and d

must therefore be proportional to d,


c, and a, respectively; it follows that a = d, and the rest is the
same as in the first solution.

Problem 5
Prove that

forallab>

(ab)2 <
8a

a+b
2

0.

Alternative

that

) -

(ab)2

8b

3. Solutions to Introductory Problems

30

i.e.

4a

4b

i.e.

(ab)2 <

< (ab)2

from which the result follows.

Solution 5, Alternative 2
Note that

(a-t-b\2
2

-ab

(a-b)2
2(a + b) +

Thus the desired inequality is equivalent to


4a

a+b+

which is evident as a b> 0 (which implies a

4b.

b).

Problem 6 [St. Petersburg 1989]


Several (at least two) nonzero numbers are written on a board. One may

erase any two numbers, say a and b, and then write the numbers a +
and b

instead.

Prove that the set of numbers on the board. after any number of the
preceding operations, cannot coincide with the initial set.

Solution 6
Let S be the sum of the squares of the numbers on the board. Note that
S increases in the first operation and does not decrease in any successive
operation, as

a)2 =

(a+
with equality only if a = b =
This completes the proof.

0.

to Introductory Problems

31

problem 7 [AIME 1986]


The

may be

written in the form

ao + aiy + a2y2 + .

+ ai6y'6 + a17y17,

are constants. Find a2.


= x + 1 and
SolutiOn 7, Alternative 1
where y

Let f(x) denote the given expression. Then

xf(x)=xx2+x3

.x18

and

(1 + x)f(x) = 1
Hence

f(x)=f(y1)=

1 (y 1)18

1 (y 1)18

l+(yl)

Therefore a2 is equal to the coefficient of y3 in the expansion of


(y

1)18,

i.e.,

a2

= 816.

Solution 7, Alternative 2
Let 1(x) denote the given expression. Then

f(x) =f(y1)

l(y l)-f(y

Thus

/2\

/3\.

(17\

Here we used the

I
and the fact that

7i

'\

/n+1"

(18"

3. Solutions to Introductory Problems

32

Problem 8
Let a. b, and c be distinct nonzero real numbers such that

-1 = b+ -1 = c+ -.1
c
b
a
Prove that

1,

Solution 8
From the given conditions it follows that
bc
ca
ab
ab=.bc,andca=.
bc
ca
ab

Multiplying the above equations gives (abc)2 =


result follows.

1,

from which the desired

Problem 9 [Putnam 1999]


Find polynomials f(s), g(x), and h(s). if they exist. such that for all x
g(')I + h(s) =

ifr<1

3x

2x+2

if 1 <

.r

ifx>0.

<

Solution 9, Alternative 1
Since x = 1 and x = 0 are the two critical values of the absolute
functions, one can suppose that

F(s) =
=

I (cab)x+da

ifs <1

if1<x<0

1(a+b+c)x+a+d ifx>0.

which implies that a 3/2, b = 5/2. c = 1. and d = 1/2.


Hence f(s) = (3s + 3)/2, g(x) = 5x/2, and h(s) = x +
Solution 9, Alternative 2
Note that if r(x) and 8(x) are any two functions, then
max(r,s)

r-l-s+lrsj

=
2
Therefore, if F(s) is the given function. we have
F(x) = max{3x 3. 0} max{5x, 0} + 3x + 2
= (3x3-'-j3x+31)/2(5x+15x1)/2-4-3x--i-2

colutions to Introductory Problems

33

Problem 10
Find all real numbers x for which
8X

12x + 18x

Solution 10

= a and

By setting

the equation becomes

b.

a3+b3 _7
a2b+ b2a
i.e.

a2ab+b2 _7
6'

ab
i.e.
Ga2

l3ab+6b2 =

0.

i.e.

(2a 3b)(3a 2b) = 0.


Therefore

2x+i = 3x+i or

2x1

x = 1.
It is easy to check that both .r =

= 3x1, which

and x =

implies that x =

such

(2n\

\n)

that

<ni

for all positive integers n.

ii
Note that

+
+

<

... +

and for n

(10)
Thus in

= 252 >

and

satisfy the given equation.

Problem 11 [Romania 1990]


Find the least positive integer in

= (1+ i)2n =

3. Solutions to Introductory Problems

34

Problem 12
Let a. b, c, d, and e be positive integers such that

abcde=a+b+c+d+e.
Find the maximum possible value of max{a, b, c. d, e}.

Solution 12, Alternative 1


Suppose that a b c d
e. We need to find the maximum
e. Since

value of

c<a+b+c+d+c<5c.

5e, i.e. < abcd 5.


Hence
= (1.1,1,2), (1,1,1,3).
(1,1, 1.5), which leads to max{e} = 5.
then e < abcdc

(1. 1,1,4), (1, 1,2,2), or

Solution 12, Alternative 2


As

before,

suppose that a

d c. Note that

de

d = 1, then a = b = c = 1 and 4 + c = e, which is impossible.


e 5.
Thusd1 land ci
It is easy to see that (1, 1,1,2,5) is a solution.
Therefore

max{e} =

Comment:

5.

The second solution can be used to determine the maxiwhen x1, x2, .
are positive integers

mum value of {x1, x2,. .


such

. ,

that

x1x2

= xi+

+ xm.

Problem 13
Evaluate
3

1!+2!+3!

2!+3!+4!

++

2001

to Introductory Problems

35

13

Note that

k+2
= k![1+k+1+(k+1)(k+2)]

k+2

k!(lc+2)

(k+2)!

(k+2)1

(k-i--2)!

(k+1)!

(k+2)!

By telescoping sum, the desired value is equal to


2

2001!

Problem 14
Find all possible values of x.

solution 14, Alternative 1


Since

\/a2 + al + 1>

al

and

2a
Va2

+a+ 1+

Va2

-a+ 1'

we have
lxi <

12a/ai

= 2,

Squaring both sides of

Va2+a+1
Yields

a + 1 = 2a x2.

3. Solutions to Introductory Problems

36

Squaring both sides of the above equation gives


4(x

1)a =x (x2'4)ora =
2

x2(x24)

4(21)'

Since a2 0, we must have


x2(x2

4)(x2 1)

0,

Since xI < 2, x2 4 < 0 which forces x2 1 < 0. Therefore, 1 < x < 1.


Conversely, for every x C (1, 1) there exists a real number a such that
x

Va2

+a+1

a+1.

Solution 14, Alternative 2


Let A =
B=

and P = (a.O). Then P


is a point on the x-axis and we are looking for all possible values of

d=PA-PB.

By the Triangle Inequality, IPA PBJ <


= 1. And it is clear
that all the values 1 <d < 1 are indeed obtainable. In fact, for such
a d, a half hyperbola of all points Q such that QA QB = d is well
defined, (Points A and B are foci of the hyperbola.)
Since line AB is parallel to the x-axis, this half hyperbola intersects the
x- axis, i.e., P is well defined.

Problem 15
Find all real numbers x for which

lOx + lix + 12x =

13X

+ 14x.

Solution 15
It is easy to check that x = 2 is a solution. We claim that it is the only
one. In fact, dividing by 13X on both sides gives

112\x
The left hand side is a decreasing function of x and the right hand side

is an increasing function of x.
Therefore their graphs can have at most one point of intersection.

Solutions to Introductory Problems

37

Comment: More generally.

a2+(a+1)2+.+(a+k)2
=(a+k+1)2+(a+k+2)2+"+(a+2k)2
forak(2k+'). kcN.
problem 16 [Korean Mathematics Competition 2001]
N be a function such that f(1, 1) = 2.
Let f: N x N

f(m + 1. n) = f(ni, n) + ni and f(772,

f(n2. n)

72 + 1)

for all rn, n c N.

Find all pairs (p, q) such that f(p. q) =

2001.

Solution 16
We have

f(pq) = f(pl,q)+p1
=
=
=
2

= f(1,l)
=

q(q1)

p(pl)

2001.

Therefore

p(p 1) q(q
2

1)

1999,

i.e.

(pq)(p+q 1) =2. 1999.


ROte

that

1999

is a prime number and thatpq < p+q 1 for p.q

We have the following two cases:

= 1999. Hence p= 1001 and q= 999.

c N

3. Solutions to Introductory Problems

38

Therefore (p. q) = (2000, 1999) or (1001,999).

Problem 17 [China 1983]


Let f be a function defined on [0, 1] such that

f(0) = f(1) = land f(a)f(b)I < labi,


bin the interval [0, 1].

for all a

Prove that
f(a) f(b)j <

Solution

1.7

We consider the following cases.


1. a

bI

< 1/2. Then If(a) f(b)f < a

- bI

as desired.

2. Ia bi > 1/2. By symmetry, we may assume that a> b. Then

f(a) f(b)I

= If(a) 1(1) + f(0) f(b)I


<
f(a) f(')I + 1(0) f(b)I
< IalI+I0bI

= 1a+b0
=

1(a-b)

as desired.

Problem 18
Find all pairs of integers (x, y) such that
+

= (x + y)2.

Solution 18
Since x3+y3 = (x+y)(x2xy+y2), all pairs of integers (nri), n C 7Z.
are solutions.
Suppose that x + y

0. Then the equation becomes


S2

i.e.

Xy + y2 = x + y,

solutions to Introductory Problems

39

a quadratic equation in x, we calculate the discriminant

A = y2 + 2y + 1 4y2 + 4y =

3y2 + 6y + 1.

solving for A 0 yields


3

Thus the possible values for y are 0, 1, and 2. which lead to the solutions

(1,0), (0,1), (1,2), (2.1), and (2,2).


Therefore, the integer solutions of the equation are (x. y) =
(1,2), (2,1), (2,2), and (n, n). for all nEZ.

(1,

0), (0, 1),

Problem 19 [Korean Mathematics Competition 2001]


Let
2
f(x)= 4X+2

for real numbers x. Evaluate

/2000

'\

f has a half-turn symmetry about point (1/2, 1/2). Indeed,


2.4x

from which it follows that 1(x)

f(1

-t-

x)

4X

1.

Thus the desired sum is equal to 1000.

Problem 20
Prove that for n 6 the equation
1

x1
has

integer solutions.

Solution 20
Note that
1

3. Solutions to Introductory Problems

40

from which it follows that if (x1, x2,

(a1

.a2,

is an inte.

ger solution to
1

x1

x2

then

(x1,x2,. .

is an integer solution to
1

xl

x2

Therefore we can construct the solutions inductively if there are solutions


for n = 6, 7, and 8.

Since xi = 1 is a solution for n = 1, (2,2,2,2) is a solution for n =


and (2, 2, 2,4,4.4,4) is a solution for n = 7.

4,

It is easy to check that (2, 2, 2,3, 3, 6) and (2, 2. 2, 3,4,4, 12, 12) are solutions for n = 6 and n = 8, respectively. This completes the proof.

Problem 21 [AIME 1988]


Find all pairs of integers (a, b) such that the polynomial
ax17 + bx'6 -i- 1

is divisible by x2 x

1.

Solution 21, Alternative 1


Let p and q be the roots of x2 x I = 0. By Vieta's theorem,
p + q = 1 and pq 1. Note that p and q must also be the roots of
ax17 + bx'6 + 1 = 0. Thus
ap17 + bp'6 =

and aq17 + bq'6

1.

Multiplying the first of these equations by q'6, the second one by p16,
and using the fact that pq = 1, we find

ap+b=q16andaq+b=p'6.
Thus

a=

q'6

(p8 + q8)(p4 + q4)(p2 + q2)(p q).

(1)

solutions to Introductory Problems

41

since

p+q =

1,

p2+q2 = (p+q)22pq=1+2=3,
p4+q4 = (p2+q2)22p2q2=92=7,
(p4+q4)22p4q4=49247,

p8+q8

follows that a 1 3 7 . 47 = 987.


Likewise, eliminating a in (1) gives

b = p'7q'7
pq

= p'6+p'5q+p'4q2++q16
=

(p'6 + q16) + pq(p'4 + q14) + p2q2 (p'2 + q'2)

+ p7q7(p2 + q2 ) + p8q8
(p16 +q'6) (p'4 +q'4)+

2n+4

+q2)+ 1.

2n+4

2n+4

P
(p2fl+2

=
for

(p2

k2 = 3 and k4 = 7, and

For n 1, let
1.

+ q2Th)

+ q2)

n 3. Then k6

18, k8 = 47, k1,, = 123, k,2 = 322, k,4

843,

k16 = 2207.
Hence

b=2207843+322123+47 18+73+1=1597
or

(a, b) = (987, 1597).

solution 21, Alternative 2


The other factor is of degree 15 and we write
(C15x15

C14X14 +

.. + cix

1)

= ax17 + bx'6 + 1.

Comparing coefficients:

x0:

c0=1.

c0c,=0,c,=1
x2:

andfor3<k<15, xk:

coc,+c2=0.c2=2,

Ck_2Ck_,+CkO.

3. Solutions to Introductory

42

It follows that for k 15,


= Fk+1 (the Fibonacci number).
Thus a =
c14 c15 = F17 = 1597
= F16 = 987 and b
(a, b) = (987, 1597).

Comment:

Combining the two methods, we obtain some interesting


and
Since

facts about sequences

it follows that
and
satisfy the same recursive relation.
easy to check that k2 = F1 + F3 and k4 = F3 + F5.
=
and
Therefore
+
I.2m

2n+1

2n2 -rLb2n4

Lf

It ,i5

L1

Problem 22 [AIME 1994]


Given a positive integer n, let p(n) be the product of the non-zero digits
of n. (If n has only one digit, then p(n) is equal to that digit.) Let

S=p(l)+p(2)+

+p(999).

What is the largest prime factor of 8?

Solution 22
positive integer less than 1000 to be a three-digit number
by prefixing Os to numbers with fewer than three digits. The sum of the
products of the digits of all such positive numbers is
Consider

each

(0O.O+OO.1+...+9.99)O.O.O
However, p(n) is the product of non-zero digits of n.

products

The sum of these

can be found by replacing 0 by 1 in the above expression, since

ignoring 0's is equivalent to thinking of them as l's in the products. (Note

that the final 0 in the above expression becomes a 1 and compensates


for the contribution of 000 after it is changed to 111.)

Hence

1 = (46 1)(462+46+ 1) = 33 5 7.103,

and the largest prime factor is 103.

Solutions to Introductory Problems

43

problem 23 [Putnam 1979]


of nonzero real numbers such that
Zn be a sequence

xn '

Xn_2Xm_1

for n = 3,4
establish necessary and sufficient conditions on x1 and x2 for
an integer for infinitely many values of n.

to be

Solution 23, Alternative 1


We have

Let

Xn_2Xn_1

= Yn1 Yn2. i.e.. y,-, is an arithmetic


is a nonzero integer when n is in an infinite set 8, the

Then

sequence. If
1.
yb's for n C S satisfy 1
Since an arithmetic sequence is unbounded unless the common difference
is 0, 7/n Yni

= 0 for all n, which in turn implies that x1 =

= m, a

nonzero integer.
Clearly, this condition is also sufficient.

Solution 23, Alternative 2


An easy induction shows that

xn_

(n

x1x2
(n

1)xi

2)x2

x1x2
(x1 x2)n + (2x2 x1)

for n

3,4
In this form we see that
of n if and only if x1 =

will be an integer for infinitely many values

= in for some nonzero integer m.

Problem 24
Solve the equation

3x

Solution 24, Alternative 1


It is clear that
1. 2

2. We consider the following cases.

x < 2. Setting x = 2cosa, 0 < a < ir, the equation becomes


8cos3 a

or

6cosa

+ 1).

3. Solutions to Introductory Problems

44

from which it follows that cos 3a =


Then 3a

or 3a + =

= 2m7r, rn

Since 0 a

cos
2n7r,

n E Z.

the solution in this case is


4ir

x=2cos0=2,
2. x> 2. Then x3 4x = x(x2

4)

>

4ir

andx=2cosT.
and

x2x2=(x2)(x4-1) >0
or

x>

It follows that

3x >

x>

Hence there are no solutions in this case.

Therefore, x = 2, x = 2cos4ir/5, and x = 2cos4ir/7.

Solution 24, Alternative 2


For x> 2. there is a real number t > 1 such that

x=t 21
The equation becomes

(t7 1)(t5

1)

which has no solutions for t> 1.


Hence there are no solutions for x > 2.
For 2 x 2, please see the first solution.

0,

Solutions to Introductory Problems

45

problem 25 [AIME 1992]


sequence of real numbers A = {ai, a2, a3,
any
For

}, define

AA

to

be

are 1, and that

Suppose that all of the terms of the sequence

=
Find al

a92 0.

SolutiOn 25
Suppose that the first term of the sequence
Then

is d.

AA={d,d+1,d+2,..}
term given by d + (n

with the

1).

Hence

with

term given by

the

=a1 +

(n 1)d+

1)(n 2).

a quadratic polynomial in ii with leading coefficient

This shows that


1/2,

Since a19 =

a92

0.

we must have
=

19)(n 92),

soa1 =(119)(1 92)/2=819.

Problem 26 [Korean Mathematics Competition 2000]


Find all real numbers x satisfying the equation
2X + 3X 4X + 6X 9X

solution 26
Setting
= a and

b,

the equation becomes

1-i-a2 -i-b2 a bab= 0.


Multiplying
Square5 gives

both sides of the last equation by 2 and completing the


(1a)2+(ab)2+(b- 1)2

3. Solutions to Introductory Problem

46

= 3X and x =

Therefore 1 =

is the only solution.

Problem 27 [China 1992]


Prove that

16<

<17.

Solution 27
Note that

Therefore

which proves the lower bound.


On the other hand.

Therefore
17.

which proves the upper bound. Our proof is complete.

Problem 28 [AHSME 1999]


Determine the number of ordered pairs of integers (m. m) for which mm
O and

in3 + n3 + 99mn =

Solution 28
Note that (m + n)3 =
333

m3

= (in + n)3 =

Hence m + n

33

+
=

m3

+ n3 + 3mn(m + n). If m + n = 33. then


n3

+ 3nin(m + n) = in3 +

is a factor of in3 + n3 + 99inn

n3

+ 99mm.

We have

99mm 333

n 33)[(m

m)2 + (m + 33)2

+ (n + 33)2]

5olution5 to Introductory Problems

Uence

47

there are 35 solutions altogether: (0,33), (1.32),

(33,0), and

(33, -33).
corn ient:

More generally, we have

a3 +

+ c3

3abc

problem 29 [Korean Mathematics Competition 2001]


Let a, b, and c be positive real numbers such that a + b c 4 and

ab+bc+ca 4.
Prove that at least two of the inequalities

abI2,
are true.

Solution 29
We have

(a+b+c)2 <16.
i.e.

a2+b2+c2+2(ab+bc+ca)<16,
i.e.

a2 + b2

-f

c2

8,

i.e.

a2+b2+c2(ab+bc+ca) 4,
i.e.

(ab)24-(bc)2+(ca)2 8,
and the desired result follows.

Problem 30
Evaluate
1

3. Solutions to Introductory

48

Solution 30
denote the desired sum. Then

Let

(2n)!

(2n)!k(n_k)!(n+k)!
n

2n

,'

(272

(2n)!

(2n)!

2(n!)

Problem 31 [Romania 1983]


Let

0 <

a < 1.

Solve

aX

for positive numbers x.

Solution 31
Taking loge yields
aX loge x =
Consider

Then

functions from

both f

f(x)g(x) =

f(x)=aX. g(x)=Iogax,

h(x)=xa.

and are decreasing and h is


has unique solution x = a.

increasing.

h(x)

Problem 32
What is the coefficient of x2 when

is expanded?

It follows that

to Introductory Problems

49

SolUtbohl 32

Let

It

= an.o + afl1X+
=
is eaSY to see that

and

Since

=
=
=
we

(1

+...) (1 + 2Thx)

1)x +

have

+
+
=

a1,2

+ 26 +

+ 22n) (22 +

=2+ 24(2272_2 1) -

22Th+2_3.2n+1 +2

1)

(2n+1 1) (2m+1

2)

Problem 33
Let in and n be distinct positive integers.

Find the maximum value of

x is a real number in the

interval (0,

Solution 33
By symmetry, we can assume that rn > n. Let y =
0 < x < 1,
< xTh and 0 < y <1. Thus

Since

Xmn) =
=
APplying the AM-GM inequality yields
I

ri

_y)m_fl
72

(n.

/
+(m

n)(1

n+mn
=

mm

3. Solutions to Introductory

50

Therefore
Is

(
=(mn)(

It \

mm

Equality holds if and only if

(rnn)y
1

or

In \
\m

Comment: For m = n + 1, we have

5n+1

<

for real numbers 0 <x <1. Equality holds if and only if x = n/(n + 1).

Problem 34
Prove that the polynomial

,
are distinct integers, cannot be written as the product of two non-constant polynomials with integer coefficients, i.e., it is

where a1, a2,

irreducible.

Solution 34
For the sake of contradiction, suppose that

is not irreducible. Let f(s) = p(x)q(x) such that p(x) and q(x) are two
polynomials with integral coefficients having degree less than n. Then

g(x) = p(x) + q(x)


is a polynomial with integral coefficients having degree less than n.
Since

p(aj)q(aj) = f(aj) = 1
and both p(aj) and q(aj) are integers,
=

to Introductory Problems

and

Thus

51

p(aj) + q(aj) =

0.

g(x) has at least ri roots. But degg <n, so g(x) = 0.


p(x)

= q(x)

and

f(x)

Then

p(x)2,

which implies that the leading coefficient of 1(x) must be a negative


integer, which is impossible, since the leading coefficient of f(x) is 1.

problem 35
Find

all ordered pairs of real numbers (x, y) for which:

and

(1+x)(1+x2)(1+x4) =

1+y7

(1+y)(1+y2)(1+y4)

1-f-x7.

35
We consider the following cases.
Solution

1. xy =

Then it

0.

clear that x =

is

0 and

(x, y) = (0,0)

is

solution.

2. xy <0. By the symmetry, we can assume that x >

0 > y. Then
(1+x)(1+x2)(1+x4) >1 and 1+y7 <1. Therearenosolutions

in this case.

3. x, y >

0 and x

y. By the symmetry, we can assume that x >

y > 0. Then
(1 +x)(1 +x2)(1 +x4) >1 +x7>1 +y7,
showing that there

4. x,y

<0

and x

are no solutions in this case.

y. By the symmetry, we can assume that x <

0. Multiplying by 1

x and 1 y

y <

the first and the second equation,

respectively, the system now reads

1x8 = (1+y7)(1x)=1x+y7xy7
1y8 = (1+x7)(1y)=1y+x7x7y.
Subtracting the first equation from the second yields

xS_yS=(x_y)-f-(x?_y7)_xy(x6_y6).
Since x

<y <0, xs

> 0.

> 0,

x y

<

0, x7

y7

<0,

xy

(1)

<0, and

Therefore, the left-hand side of (1) is positive while


the right-hand side of (1) is negative.

Thus

there are no solutions in this case.

3. Solutions to Introductory Problems

52

5.

x=

y.

Then solving
1

leads

to x =

0,

xy7

1 x + X7

X8

1, 1, which implies that (x, y) = (0.0) or (1, 1).

Therefore, (x, y) = (0,0) and (1, 1) are the only solutions to the
system.

Problem 36
Solve the equation

1)x2

(2x2 2)x

2x+1 2

for real numbers x.

Solution 36
Rearranging terms by powers of 2 yields

1)2(x2+x1) =0.
Setting y =

(1)

and dividing by 2 on the both sides, (1) becomes

2Yx+2xy_(x+y) =0
or
(2)

Since f(x) =

and x always have the same sign,

0.
Hence if the terms on the left-hand side of (2) are nonzero, they must
have the same sign, which in turn implies that their sum is not equal to
0.

Therefore (2) is true if and only if x =

x = 1,0, and

or y =

0.

which leads to solutions

1.

Problem 37
Let a be an irrational number and let n be an integer greater than 1.
Prove that
(a+ Va2_
+ (ais

an irrational number.

to Introductory Problems

53

37

SolUtbol1

Let

.1.

N= (a+

+ (a

and let

1)*,

For the sake of contradiction, assume that N


b + 1/b.
Then by using the identity

Then N =
rational.

+
repeatedly for m
m E N.
In particular,

1)

we obtain that btm + 1/btm is rational for all

1, 2

=a+

1+a- Va2 - 1=2a

is rational, in contradiction with the hypothesis.


Therefore our assumption is wrong and N is irrational.

Problem 38
Solve the system of equations
(x1

X2

x3)2

x2(x4 + X5 x2)

(x2x3+x4)2 = x3(x5+x1x3)
(x3x4+x5)2

=
=
=

(x5x1-f-x2)2
for real numbers x1, x2,

x4(x1+x2x4)
x5(x2+x3x5)

xi(x3+x4x1)

x3, x4, S5.

38
Let 5k+5 =

Adding the five equations gives

is

+ 2XkXk+2) =

+ 2XkXk+2).
k=1

It follow5 that

= 0.

3. Solutions to Introductory Problems

54

Multiplying both sides by 2 and completing the squares yields

from which x1 =
system are

x2

X3

x4

= 0,

Xk+1)2

Therefore the solutions to the

(xi,x2.x3,x4,xs) =

(a, a, a. a, a)

for a C R.

Problem 39
Let x. y, and z be complex numbers such that x+y+z =

2,

x2+y2+z2

3, and xyz = 4.
Evaluate
1

xy+z1

yz+xl

zx+yl

Solution 39
Let S be the desired value. Note that

xy+zl =xy+1x--y=(xl)(y1).
Likewise,

yz+xl

(y 1)(x 1)

zx+y 1 =

(z 1)(x 1).

and
Hence
1

(x_1)(y_1)+(y_1)(z_1)+(z_1)(x_1)

1
x+y+z3
(x-l)(y-l)(z--l)
1

xyz(xy+yz+zx)+x+y+z--

5(xy+yz+zx)

But
2(xy + yz + zx) =

Therefore S = 2/9.

(x

+ y

z)2

(x2

+ z2) =

1.

5oiutions to Introductory Problems

55

problem 40 [USSR 1990]


Mr Fat is going to pick three non-zero real numbers and Mr. Taf is going
8rrange the three numbers as the coefficients of a quadratic equation

x2+ x+ =0.
Mr. Fat wins the game if and only if the resulting equation has two
distinct rational solutions.
Who has a winning strategy?

Solution 40
Mr. Fat has the winning strategy. A set of three distinct rational nonzero
numbers a, b, and c, such that a + b + c = 0, will do the trick. Let A, B,
and C be any arrangement of a, b, and c, and let f(x) = Ax2 + Bx + C.
Then

f(1) = A

+ B + C = a+b + c

=0.

which implies that 1 is a solution.


Since the product of the two solutions is C/A, the other solution is C/A.
and it is different from 1.

Problem 41 [USAMO 1978]


Given that the real numbers a, b, c, d,

and e

satisfy

simultaneously the

relations

a+b+c+d+e=Sanda2+b2+c2+d2+c2= 16,
determine the maximum and the minimum value of a.

Solution 41, Alternative 1


Since the total of b.c. d.

and

c is S a, their average is x =

a)/4.

(S

Let

b=x+b1, c=x+c1, d=x+d1, e=x+c1.


Then

b1

+ d1 + c1 =

and
(8a)2

(1)

or

0 5a2

16a

= a(5a

16).

Therefore 0 a 16/5, where a = 0 if and only if b =


nd a 16/5 if and only if
cd
6/5

3. Solutions to Introductory Problem5

56

Solution 41, Alternative 2


By the EtMS-AM inequality, (1) follows from

and the rest of the solution is the same.

Problem 42
Find the real zeros of the polynomial
Pa(X) = (x2 + 1)(x
where

1)2

ax2.

2x + 1) ax2

0.

1'\
x + I (x 2 + I a

0.

a is a given real number.

Solution 42
We have

(x2 + 1)(x2

Dividing by x2 yields

I
I

1\I

xj\

xj

By setting y = x + 1/x, the last equation becomes


y2

0.

It follows that

x+- =
x

which in turn implies that, if a 0, then the polynomial Pa(X') has the
real zeros

1+

In addition, if a 8, then Pa(X) also has the real zeros

1- fiPi

Problem 43
Prove that
1

for all positive integers n.

2n1
2n

Solutions to Introductory Problems

57

We use induction

For n = 1, the result is evident.


SuPPOSe the statement is true for some positive integer k, i.e.,

2ki

2k

Then

2k1 2k+1

2k+1

2k

In order for the induction step to pass it suffices to prove that

2k+1

2k+2 < y3k+4


This reduces to

(2k+1\2

3k+1

<3k+4'
i.e.

(4k2+4k+1)(3k+4) <(4k2+8k+4)(3k+1),
i.e.

0< k,
which is evident. Our proof is complete.

Comment:
proved to

By using Stirling numbers, the upper bound can be imfor sufficiently large n.

Problem 44 [USAMO Proposal, Gerald Heuer]


Let

P(x) =
a nonzero

polynomial with integer coefficients such that

P(r) =
for Some integers

P(.s)

r and s, with 0 <r <s.

that ak s for some k.

3. Solutions to Introductory

58

Solution 44
Write P(x) = (x S)XCQ(X) and

+ ... +

Q(x) = b0xm +

0. Since Q has a positive root, by Descartes' rule of signs,


either there must exist some k for which bk > 0 bk4l, or bm > 0.
If there exists a k for which bk > 0 bk+1, then
where bm

ak+1 = sbk

+ bk+1 S.

If bm > 0, then
S.
= sbm
In either case, there is a k such that ak s, as desired.

Problem 45
Let rn be a given real number, Find all complex numbers x such that
'.2
'.2
/
/
IX
IX
\
\
+I1 =772
I1
\x+1J \X1J +712.
2

Solution 45
Completing the square gives

\2
2x2
xlj = x 21 +m2+rn,

/ x
\,x+l
i.e.

2x2

\2
I

2x2

x21

+712 +772.

Setting y = 2x2/(x2 1), the above equation becomes

(rn2

+ m) = 0,

i.e.

(ym1)(y+m)=0.
Thus
2x2

x21

=772or

2x2

x21

=712+1,

which leads to solutions

-2andx

1.

solutions to Introductory Problems

59

problem 46

The sequence given by xo = a,

b,

and

2\

js periodic.

prove that ab =

1.

solution 46
on both sides of the given recursive relation yields

by

+1

or

1)

1.

is a geometric
1 for n C N. Since Yn+i =
xn is periodic, then so is
which implies that
= 0 for
all n E N. Therefore

=
Let
sequence.

If

ab =

XjXl

Yi

+ 1=

1.

Problem 47
Let a, b, c, and d be real numbers such that
(a2 + b2

1)(c2

d2

1) >

(ac+ bd

1)2.

Prove that
a2

+ b2> I and c2 + d2 >

1.

Solution 47
For the sake of the contradiction, suppose that one of a2 + b2 or c2 + d2
'S less than or equal to 1. Since (ac+bd 1)2
0, a2 +b2 land
C2 + d2 1
must have the same sign. Thus both a2 b2 and c2 + d2 are
less than 1. Let

x=1a2b2andy=1c2---d2.
Thefl 0 < x, y

4xy

1. Multiplying by 4 on both sides of the given inequality

+ 2bd 2)2 = (2 2ac 2bd)2


= (a2+b2+x-i-c2+d2+y2ac2bd)2
= [(ac)2
>

(2ac

(x+y)2=x2+2xy+y2,

3. Solutions to Introductory Problems

60

or 0> x2 2xy f y2 = (x

y)2, which is irhpossible.

Thus our assumption is wrong and both

a2 + b2

and

c2

+ d2 are

than 1.

Problem 48
Find all complex numbers 2 such that

(3z+1)(4z+

1)(6z+ 1)(12z+ 1) =2.

Solution 48
Note that

8(3z

+ 1)6(4z + 1)4(6z 4- l)2(12z -1- 1)

768,

i.e.

+4)(24z+2) = 768.

(24z+8)(24z +6)(24z
Setting u

24z

+ 5 and w =

u2

yields

1)(u 1)(u3).= 768.

(u+3)(u+
i.e.

(u2

1)(u2 9)

i.e.

lOw

759

0,

i.e.

(w33)(w+23)

Therefore

the

solutions

to the

24

=0.

given equation are

andz=
24

Problem 49
Let x1,x2,

P(x) =
Prove that

be the zeros different from


1,

of the polynomial

2.

1x1

1x2

ni

1-

to Introductory Problems

61

Alternative 1

5olutiofl

Q(x) = P(1 x) =

(1

Then

Q(x) = (_1)nxnl +
and

+ +

are the nonzero roots of the polynomial Q(x). as

(1ai

=0.

Thus the desired sum is the sum of .the reciprocals of the roots of polynomial Q(x), that is,
1
1

X1

=++4-
a2
1

a1

= a2a3
By

+ ala3

--

+ a1a2

the Vieta's Theorem, the ratio between

and

is equal to the additive inverse of the ratio between the coefficient of x


and the constant term in Q(x), i.e., the desired value is equal to

(n)

as desired.

Solution 49, Alternative 2


For any polynomial R(x) of degree ni, whose zeros are Xi.

the following identity holds:


1

XX1

+...+

R'(x)
R(x)

Xn_ 1'

3. So'utions to Introductory Problems

62

For

Xl

x1

+...+x+1,

R(1) = n and

R'(i)=(n 1)+(n2)+'+ =

n(n

1)

It follows that
1

1x1

+...+
1x2

R'(l)

ni

R(1)

Problem 50
Let a and b be given real numbers.
Solve the system of equations

s/f x2 + y2
for real numbers x and y.

Solution 50
Let u = x,+ y and v = x

y.

Then

U+V

UV

Adding the two equations and subtracting the two equations in the original system yields the new system

(a+b)s/1uv

=
=

Multiplying the above two equations yields


uv(1 uv)

hence uv =

a2

b2.

(a2

b2)(1

uv).

It follows that

u= (a+b)i/1a2+b2 andv= (ab)V1a2+b2


which in turn implies that

(a+bV'a2_b2 b+a'1a2b2
-i-b2

whenever 0 < a2

b2

< 1.

SOLUTiONS TO
ADVANCED PROBLEMS

4. SOLUTIONS TO
ADVANCED PROBLEMS
problem 51
f2000\

f2000\

1 2000\

(2000

Solution 51
Let

2000

1(x) = (1 + x)200 =
Let w = (1

Then w3 = 1 and

((2000\

(2000\

= 0. Hence

(2000

= f(1) + wf(w) + w2f(w2)


= 22000 +
+
+

w2)200

= 22000 + w(w2)200 + w2(w)200

=
Thus

22000

= 22000 1.

the desired value is

22000

Problem 52
Let x, y, z be positive real numbers such that x4 + y4 + z4 =
Determine with proof the minimum value of
1 x8

52
For 0 < u < 1, let 1(u) = u(1
RY the AM-GM inequality,

1 y8

u8).

1 z8

Let A be a positive real number.

A(f(u))8 =Au8(1 _u8)..(1 _u8) <

[Au8 +8(1 _u8)]9

4. Solutions to Advanced

66

Setting A =

in the above inequality yields

8(f(u))8 <

(8)

or

It follows that
1 x8

+
1

y8

x(1

x8)

y(1

y8)

z(1 z8)

>
8

with equality if and only if


x=y

Comment: This is a simple application of the result of problem 33 in


the previous chapter.

Problem 53 [Romania 1990]


Find all real solutions to the equation
2X + 3X + 6X

x2.

Solution 53

x < 0, the function f(x) = + + x2 is increasing, so the


equation 1(x) = 0 has the unique solution x = 1.
Assume that there is a solution s 0. Then
For

so s
But then s

and hence
LsJ

3S + 6S

3,

1.

yields

2S 2LsJ = (1 +

which in turn implies that


GS > 4S = (23)2

I+

s,

_____________

Solutions to Advanced Problems


28

+
x

67

> s2, a contradiction.


1 is the only solution to the equation.

problem 54
j4et {an}n1 be a sequence such that al =

and

2
1

=
for all n E N.

Find an explicit formula for

Solution 54
Solving the equation

Note

leads to x

that

Therefore,

2+1)

and

+i]
1

Problem 55
Let x, y, and z be positive real numbers. Prove that

y+f(y+z)(y+x)
1.

55
Note

that

.J(x+y)(x+z)

4. Solutions to Advanced Problems

68

In fact, squaring both sides of the above inequality yields

x2+yz
which

is evident by the AM-GM inequality. Thus


x

Likewise,
v'V

and
z

<

Adding the last three inequalities leads to the desired result.

Problem 56
Find, with proof, all nonzero polynomials f(z) such that

f(z2) + f(z)f(z +

1)

0.

Solution 56
Let

1(z) =

azm(z

where m and n are non-negative integers

and
g(z) = (z zi)(z

0 and ;

1, for i = 1,2,..

az2m(z

. ,

k. The given condition becomes

1)m(z

Zk),

zi)(z2

(z2 Zk)

z2)...(z zk)

.(z+1z1)(z+ 1z2)(z+1zk).
a=

and f is nonzero, so a =
that is, m = n.
Thus f is of the form
Thus

a2,

1.

Since z2

1, 1 z2

Then 22m =

zm(z 1)mg(z).

Dividing by Z2m(2

the last equation becomes

g(z2) = g(z)g(z + 1).

0.

,lutions to Advanced Problems

claim that g(z)


one complex root r
We

69

1. Suppose not; then clearly g must have at least


0. Now

= g(r)g(r +
= 0,
= 0.

g(r2)
g(r4)
g(r8)

1)

0,

and so on.
Since g cannot have infinitely many roots, all its roots must have absolute
value 1.
Now,

g((r 1)2) = g(r

soj(r1)21 =

1)g(r) =

0,

1.

Clearly, if

then
2

'

But r2 is also a root of g, so the same should be true of r2:


7'

fi+vi
2

This is absurd. Hence, g cannot have any roots, and g(z)


Therefore, the f(z) are all the polynomials of the form

for

mEN,

Problem 57
Letf: N * N be a function such that f(n+ 1) > f(n') and f(f(n)) = 3n
for all ri.

Evaluate f(2001).

57, Alternative 1
We prove the following lemma.

Lemma For ri=0,1,2,...,


:1.

2.

and

4. Solutions to Advanced Problems

70

We use induction.

Proof

For n = 0, note that f(1) 1, otherwise 3 = f(f(1)) = 1(1) = 1, which


N, f(1) > 1. Since f(n + 1) > 1(n),
is impossible. Since f N
f is increasing. Thus 1 < 1(1) < f(f(1)) = 3 or 1(1) = 2. Hence
I (2) = 1 (1 (1)) = 3.
Suppose that for some positive integer ri 1,

f
and

f (f(3n+1))

desired. This completes the induction.


There are
1 integers m such that
3fl 1 integers m' such that

= 3n+2,

as

< in <

and there are

f
2

for 0 m 3n

3fl

+in,

Therefore

+m) =f(f (3fl +m)) =


Hence

1(2001) = 1(2

36

+ 543) =

(36

+ 543) =

3816.

Solution 57, Alternative 2


For integer n, let fl(3) =

denote the base 3 representation of

a1a2

Using similar inductions as in the first solution, we can prove that

ifa1 =1.

1a2

if al =

2.

Since 2001(3) = 2202010, f(2001)(3) = 12020100 or


1 (2001) = 1 32

+2

34

+2

36

37

3816.

Advanced Problems

71

problem 58 [China 1999]


Let F be the set of all polynomials f(x) with integers coefficients such
f(x) = 1 has at least one integer root.
the least integer greater than 1 for which
For each integer k> 1, find
F
such
that
1(x)
= mk has exactly k distinct integer
there exists an f E
roots.

Solution 58
has exactly
Suppose that fk E F satisfies the condition that 1k (x) =
k distinct integer roots, and let a be an integer such that fk(a) = 1. Let

be the polynomial in F such that


gk(X)
for

fk(x+a)

all x.

= fk(a) = 1, so the constant term of is 1. Now gk(x) = mk


exactly
k distinct integer roots r1, r2
rk, so we can write
has
Now

gk(x)
where qk(x) is

an integer polynomial.

Note that r1r2


equals 1- mk.
Since rnk > 1,

mk = (x r1)(x r2) ... (x

. Tk

divides the constant term of gk(x)

1 172k cannot
1

Now

are

mk, which

be 0,

mk! rir2

distinct integers, and none of them isO. so

rlr2..rkl
hence

mk
Thi5 value of mk
gk(x)

is

Ik/21! + 1.

attained by

1)(x+1)(x-2)(x+2)
(x+ (_1)klk/2]) + Lk/2i! [k/2]! + 1.

Thus,

mk = Lk/2Th [k/2]! + 1.

4. Solutions to Advanced

72

Problem 59
Let x1 = 2 and
=

Xi

X2

22

xn

Solution 59
Since x1 =

and

1),

is increasing.
Then
1
0.

_1

Hence

1)

or

+...+=1x2
x1
1

Thus it suffices to prove that, for m


1

22

xn+11

N,
1

<1_

22

or

(1)

We use induction to prove (1).


For ri = 1, X2 =
x1 + 1 = 3 and (1) becomes 2 < 3 <4, which is
true.
Now suppose that (1) is true for some positive integer ii = k, i.e.,
22k1

22k

(2)

Then for ri =

k + 1,

Xk+2

the lower bound of (1) follows from

1)

22k1

22k

Solutions to Advanced Problems


Since xk4-1 is

73

an integer, the lower bound of (2) implies that


xk+1

22k

and

1,

from which it follows that


xk+2

22k

Xk+1(Xk+1

(22k

1)

i)

22k+1

desired.

This finishes the induction and we are done.

problem 60 [Iran 1997]


R+ is a decreasing function such that for all
Suppose that f
:

z,y E R+,

f(x +

y)

+ f(f(x) + 1(y)) =

f(f(x + f(y)) + f(y + 1(x))).

Prove that f(f(x)) = x.


Solution 60
Setting y = x gives

f(2x) + f(2f(x)) = f(2f(x + f(x))).


Replacing x with f(x) yields

f(2f(x)) + f(2f(f(x))) = 1(21(1(x) + f(f(x)))).


Subtracting these two equations gives

1(21(1(x))) - f(2x) = f(2f(f(x) + f(f(x)))) - f(2f(x + 1(x))).


If 1(1(x)) > x, the left hand side of this equation is negative, so

f(f(x) +1(1(x))> f(x+ f(x))


and

f(x)+f(f(x)) <x 1-f(x),


contradiction A similar contradiction occurs if f(f(x)) <x.
Thus f(f(x)) = x as desired.
a

Comment: In the original formulation I was meant to be a continous


function. The solution above shows that this condition is not necessary.

Solutions to Advanced

74

Problem 61 [Nordic Contest 1998]


Find all functions f : Q i Q such that

f(x+y)+f(x -y)2f(x)+2f(y)
for all x,y EQ.
Solution 61
The only such functions are 1(x) = kx2 for rational k. Any such function
works, since

f(x+y)+f(xy)=k(x+y)2+k(x--y)2
=kx2+2kxy+ky2+kx2 2kxy+ky2
= 2kx2 + 2ky2
= 21(x) + 21(y).

Now suppose f is any function satisfying

f(x+ y)+ f(xy) =2f(x)+2f(y).


Then letting x = y = 0 gives 21(0) = 41(0), so 1(0) = 0.
We will prove by induction that f(nz) = n2f(z) for any positive integer
ri and any rational number z.
The claim holds for ri = 0 and ri = 1; let ri 2 and suppose the claim
holds for n

and n 2.
Then letting x = (n 1)z, y
1

z in the given equation we obtain

f(riz) + f((n 2)z) = f((n

1)z

+ z) + f((n 1)z z)

=2f((n1)z)+2f(z)
so

f(nz) = 2f((ri 1)z) + 2f(z) f((ri 2)z)


= 2(n 1)21(z) + 2f(z) (n 2)21(z)
(2n24n+2+2n2+4n4)f(z)
= ri2f(z)
and the claim holds by induction.

Letting x =

in the given equation gives

f(y)
so

n.

f(y) = 1(y)

for

+ f(y) =

2f(0) + 2f(y)

all rational y; thus f(nz) = n2f(z)

for

all integers

Solutions to Advanced Problems

Now

75

let Ic = f(1): then for any rational number x = p/q,

q2f(x) = f(qx) = 1(p) = p2f(1) =


5

Thus

kp2

1(x) = kp2/q2 = kx2.

the functions f(x) = kx2.

k E

Q. are the only solutions.

Problem 62 [Korean Mathematics Competition 2000]


Let

<a < 1.

Prove

that the equation

x3(x+1)=(x-i-a)(2x--a)
has four distinct real solutions and find these solutions in explicit form.

Solution 62
Look at the given equation as a quadratic equation in a:
a2+ 3xa+2x2 x3 x4 =0.
The discriminant of this equation is
9x2 8x2

+ 4x3 + 4x4 = (x + 2x2)2.

Thus

The first choice a =


whose solutions are

3x(x+2x2)
2

+ x2 yields the quadratic equation x2 x a =

(1'/1+4a)
2

The second choice a =

2x

x2

yields the quadratic equation

x2 + 2x + a =
whose solutions are

The inequalities

Show that the four solutions are distinct.

0,

0.

4. Solutions to Advanced

76

Indeed

reduces to

< 3 v'l + 4a
which is equivalent to

6V1 + 4a < 6 + 8a,

or 3a <

4a2,

which is evident.

Problem 63 [Thurnament of Towns 1997]


Let a, b, and c be positive real numbers such that abc =
Prove that
1
1
1
<1.
+
+

a+b+1

1.

c+a+1

b+c+1

Solution 63, Alternative 1


Setting x =

+ b, y = b + c and z = c + a, the inequality becomes

x+1 +
y+l+ <1,
z+1
1

i.e.

i.e.

y+l

z+1

y+z+2
(y+1)(z+1)

x+1'
<

x+1'

i.e.

xy+xz+2x+y+z+2 xyz+xy+xz+x,
i.e.

x+y+z+2 xyz,
i.e.

2(a+b+c)+2

(a+b)(b+c)(c-4-a),

i.e.

2(a + b + c) a2b + ab2 + b2c + be2

c2a + ca2.

By the AM-GM inequality,

(a2b-t-a2c+1)

3a.

Advanced Problems

Likewise,

77

(b2c+b2a+ 1) 3b
(c2a+c2b+ 1) 3c.

Therefore we only need to prove that

2(a+b+c)+3 3(a+b+c),
i.e.

3 a+b

+ c,

which is evident from AM-GM inequality and abc =

1.

Solution 63, Alternative 2


b=

Let a =

Then a1b1c1 =

1.

Note that

0,

(ai
which implies that

a1b1(a1+bi).
Therefore,
1

a+b+1 =
1

aibi(ai + b1) +a1b1c1


a1b1c1

aibi(ai

+ b1 + c1)

Cl

a1

b1

a1+b1+c1
Adding

the three inequalities yields the desired result.

4. Solutions to Advanced Problems

78

Problem 64 [AIME 1988]


Find all functions f, defined on
gers,

the set of ordered pairs of positive inte..

satisfying the following properties:

f(x,x)x, f(x,y)=f(y.x). (x+y)f(x.y)=yf(x,x-4-y).


Solution 64
We claim that f(x. y) = lcm(x. y). the least common multiple of x and
y. It is clear that
lcm(x, x) = x
and

lcm(x, y) = lcm(y, x).

Note that

lcm(x.y)=

gcd(x.y)

and

gcd(x.y) =gcd(x,x+y),
where gcd (u.

v) denotes

the greatest common divisor of u

(x+y)lcm(x,y)

and

v.

Then

x(x+y)
gcd(x,x+y)

= ylcm(x,x+y).
Now we prove that there is only one function satisfying the conditions of
the problem.

For the sake of contradiction, assume that there is another function


g(x, y) also satisfying the given conditions.
Let S be the set of all pairs of positive integers (x, y) such that f(x, y)
g(x, y), and let (in, ii) be such a pair with minimal sum m+ri. It is clear
that in n, otherwise

I (in, ii) = f(m. m) = in = g(in. in)

= g(ni. n).

By symmetry (f(x, y) = f(y, x)), we can assume that n in >

Note that
nf(rn, n in)

= [in + (n m)]f(rn. ii in)


= (m ni)f (in, rn + (n in))
=

(71. in)f(m., n)

0.

Solutions to Advanced Problems

or

f(rn.rirn)=

[,ikewiSe,

g(rn, n in)

79

nrn

f(rnn).

= ii
.

g(rn. n).

Since f(rn. ii) g(m. n), f(rn. n in) g(rn. n in).


Thus (in. n rn) E S.
But (in. n in) has a smaller sum in + (n in) = n. a contradiction.

Therefore our assumption is wrong and f(x, y)

lcm(x. y) is the only

solution.

problem 65 [Romania 1990]


ii complex numbers zk. such that IzkI 1. k = 1.2
Prove that there exist c1.e2, . ..
E (1. 1} such that, for any in
Consider

+ e2z2 +

ii,

+ emzrnl 2.

Solution 65
Call a finite sequence of complex numbers each with absolute value not
exceeding 1 a green sequence.
Call a green sequence
happy if it has a friend sequence
of is and is, satisfying the condition of the problem.
We will prove by induction on ii. that all green sequences are happy.
For n = 2. this claim is obviously true.

Suppose this claim is true when n equals some number in. For the case
rn + 1. think of the zk as points in the complex plane.
For each k. let ek be the line through the origin and the point corresponding to zk. Among the lines
some two are within 600 of
each other; suppose they are
and
with the leftover one being 4.
The fact that
and
are within 600 of each other implies that there
exists some number c9 {1, 1} such that z' =
+ e9z9 has absolute
value at most 1.
the sequence z'.
is a k-term green sequence. so,
z4. z5
by the induction hypothesis, it must be happy: let c'.
C4. C5
Ck+1
of ii =

be

its friend.

Let Ca

1.

Then the sequence


comp'ete

is the friend of

Induction is now

4. Solutions to Advanced Problems

80

Problem 66 [ARML 1997]


Find a triple of rational

(a, b, c) such that

Solution 66
Let x =

and y =

Then y3 =

and x =

Note that

1 =y3 1 =(y 1)(y2+y+ 1).


and
2

x3 =yl=

(y+l)3

or

(1)

y+1
On the other hand,

3=y3+1=(y+1)(y2y+1)
from which it follows that
1

(2)

y+l
Combining

L1

(1) and (2), we obtain

x=
Consequently,

is

a desired triple.

(4

Solutions to Advanced Problems

81

problem 67 [Romania 1984)


Find the minimum of

(X2

are real numbers in the interval

where Xl.x2

1).

Solution 67
Since logs x is a decreasing function of x when 0 < a <
(x 1/2)2 0 implies x2 x 1/4, we have

xk+1

and. since

It follows that

+
2

(logx2

(X3

logx3
+
logx2

logxi

2n

by the AM-GM inequality.


Equalities hold if and only if

Problem 68 [AIME 1984]


Determine x2 + y2

22_12

z2

+ w2 if

+2232+2252 +22_72 =1,

42_ 12 + 42_32 + 42_52 + 42_72 =

62_12+62_32+
82_12

z2
62

52+62

w2
72

1,

1,

+8232+8252+8272=1.

4. Solutions to Advanced

82

Solution 68
The claim that the given system of equations is satisfied by
and w2 is equivalent to claiming that
z2

w2

t_12+t_32+i_52+t_72_l

(1)

satisfied by t = 4, 16. 36, and 64.


Multiplying to clear fractions, we find that for all values of t for which it
is defined (i.e., t 1, 9,25, and 49), (1) is equivalent to the polynomial
equation
is

P(t) =

0,

where

P(t) =

(t 1)(t

9)(t 25)(t 49)


25)(t 49) y2(t 1)(t 25)(t

x2(t 9)(t

z2(i

Since deg P(t) =

4,

l)(t

9)(t

49)

w2(t

1)(t

P(t) = 0 has exactly four zeros t =

9)(t
4,

49)

25).

16, 36, and 64,

i.e.,

P(t) =

(t

4)(t

16)(t

36)(t

64).

Comparing the coefficients of t3 in the two expressions of P(t) yields

I +9+25+49+x2+y2+z2+w2 =4+16+36+64,
from which it follows that
+ y2 + z2 + w2 =

36.

Problem 69 [Balkan 1997]


Find all functions f : R

IR

such that

f(xf(x) + 1(y)) = (f(x))2 + y


for all x,y ER.

Solution 69
Let f(0) = a. Setting x =

in the given condition yields

I (f (y)) = a2 +
for all y E R.
Since the range of a2 +y consists of all real numbers, f must be surjective.

Solutions to Advanced Problems

83

there exists b C R such that f(b) = 0.


getting x = b in the given condition yields
Thus

f(f(y)) = f(bf(b) + 1(y)) =


for

(1(b))2

+y=

all y C IL It follows that, for all x, y

(f(x))2 + y = f(xf(x) +
f [1 (1 (x))f(x) + f(y)] = I [f(x)f(f (x)) +
= f(f(x))2 + y = x2 + y,
that is,

yJ

(f(x))2=x2.

(1)

It is clear that f(x) = x is a function satisfying the given condition.


Suppose that f(x) x. Then there exists some nonzero real number c
such that f(c) = c. Setting x = cf(c) + f(y) in (1) yields
[f(cf(c) + 1(y))]2 = [cf(c) + 1(y)]2 =
for all y C R, and, setting x =

fc2

+ 1(y)]2,

in the given condition yields

f(cf(c)+f(y)) = (f(c))2+y =c2+y,


for all y

C IL

that (f(y))2 =

y2.

It follows that
fc2 +1(y)]2 =

(c2

+y)2,

or

f(y)=y,
for all y C R, a function which satisfies the given condition.

Therefore the only functions to satisfy the given condition are 1(x) = x
or 1(x) = x, for x C R.

Problem 70
The numbers 1000,
,2999 have been written on a board.
Each time, one is allowed to erase two numbers, say, a and b, and replace

them by the number

min(a, b)

After 1999 such operations, one obtains exactly one number c on the
board.

Prove that c <

1.

4. Solutions to Advanced

84

Solution 70
By symmetry, we may assume a b. Then

1.

11

We have

from which it follows that the sum of the reciprocals of all the numbers
on the board is nondecreasing (i.e.. the sum is a monovariant).
At the beginning this sum is

s-++...+<-.
1000
1001
2999 c
1

where

1/c is the sum at the end. Note that, for I


1

2000k

11
->
c 1000
1

4000

4000

i\
2999)

as desired.

Problem 71 [Bulgaria 1998]


be real numbers, not all zero.
Prove that the equation

Let a1, a2

has at most one nonzero real root.

Solution 71
Notice that

jij1jfJ

/1

i,

>
1,

999,

2000+k 20002k2 > 20002

Rearranging terms in S yields

or c <

+ a2x is concave. Hence

2998)

45olutions to Advanced Problems


is

85

concave.

Since f'(x) exists. there can be at most one point on the curve y = f(x)
with derivative 0.
Suppose there is more than one nonzero root.

Since x = 0 is also a root, we have three real roots x1 < x2 < x3. Applying the Mean-Value theorem to 1(x) on intervals [xl.X2] and [x2,x3],
we can find two distinct points on the curve with derivative 0, a contradiction.

Therefore, our assumption is wrong and there can be at most one nonzero

real root for the equation f(x) =

n.

Problem 72 [Turkey 1998]


be the sequence of real numbers defined by a1 =

Let

1.

for n
For

t and

how many distinct values of t do we have a1998 =

0?

Solution 72, Alternative 1


Let 1(x) = 4x(1

x). Observe that

= {0. 1},

f1(1) = {1/2}.

and

1])

= [0,1],

2n_1

[0,1).

= {x E R:

Let

= 0}; then

=
=
We claim that for alln 1,

For n

[0. 1]. 1 E

+ 1.

and

1. we have

A1 = {x E R f(x) = 0} =
I

fO. 1},

and the claims hold.


NOW

8o

suppose n 1 and

[0. 1].

C [0. 1]. 1 e

and

+ 1. Then

4. Solutions to Advanced

86
Since

= 0 for all n 1, so I e

f(O) = f(1) = 0, we have

Now we have

=
=
aE

{x:f(x)=l}I+

1+

I{x:f(x)=a}I

>

2
aE
aE[O,1)

=
=
=

1+2(2n_1+1_1)

Thus the claim holds by induction.


Finally, a1998 = 0 if and only if I 1997(t) =
values of t.

0,

so there are 21996 + 1 such

Solution 72, Alternative 2


As in the previous solution, observe that if f(x) E
so if a1998 = 0 we must have t E [0. 1].
Observe that for any

f(sin2

(1

sin2 0.

a2 =

= 4sin2

sin2 20.

a1998 =

a3 =

a1ggg = sin2 219970.

sin2 40

sin 219970 = 0

7Z.

Thus the values of t which give a1998 =

are

sin2 (kir/21997),
k

sin2

it follows that

Therefore

for some k e

1] then x e

ir/2[ such that sin U =

Now choose U e

since

[0,

Z, giving 21996 + 1 such values of t.

[0, 1],

to Advanced Problems

problem 73 [IMO 1997 short list]


(a) Do there exist functions 1 : R p R and g :

f(g(x))=x2

such that

such that

and

for all x e
(b)

Do there exist functions f R i R and g R

f(g(x)) = x2

g(f(x)) =

and

for all x E R?

Solution 73

(a) The conditions imply that f(x3) = f(g(f(x))) = [1(x)]2, whence


XE fi, 0, 1}

x3 = x

f(x) =

f(x) E

{O, 1}.

Thus, there exist different a, b E fI, 0, 1} such that 1(a) = 1(b).


But then a3 = g(f(a)) = g(f(b)) = b3, a contradiction.

Therefore, the desired functions f and p do not exist.


(b) Let
p(x) =

if lxi 1

I
1

- In IxI if 0 < xi < I


ifx=0.

Note that g is even and al = bi whenever g(a) =


are

g(b); thus, we

allowed to define f as an even function such that

where y is such that p(+y)


= x.

1(x) =

We claim that the functions f, p described above satisfy the conditions of the problem.

It is clear from the definition of f that f(p(x)) = x2.


Now let y =
Then g(y) = x and

p(f(x)) = 9(y2)
(y2)Ifl(Y2)

y4Ifl Y =

= (yInY)4
(

=
=

[9(y)]4
x4.

if

if 0< y < 1

ify=0

4. Solutions to Advanced Problems

88

Problem 74 [Weichao Wu]

LetO< a1 a2

Suppose

<b1

that there exists 1 k

n such that

a, for 1

k and

for 1> k.
b2
Prove that
a1a2

Solution 74, Alternative 1


We define two new sequences. For i = 1.2

and

n, Let

Then
/

ak

= ak = (a1

or

(ak

Therefore

nak =

a2 +

+ ... +

Applying the AM-GM inequality yields

(bib2.bnafl*

from which the desired result follows.

Solution 74, Alternative 2


We define two new sequences. For =

1.2

n, Let

and
Then
(1)

Note that, for cy(x

y)(y + c) 0

x+c x>yandc>0:
yy+c,

>---.

45olutions to Advanced Problems


Setting

x=

aj/bi

Using

y=
and c = ak
the above inequality implies that
for i = 1. 2, , n. Thus,
.

(1) and the AM-GM inequality yields


)

+... +
.

or

aia2

It is clear that the desired result follows from (2) and (3).

Problem 75
Given eight non-zero real numbers a1, a2
a8, prove that at least one
of the following six numbers: a1a3 + a2a4, aja5 + a2as, a1a7 + a2a8,
a3a5 + a4a5, a3a7 + a4a8,

+ a6a8 is non-negative.

Solution 75 [Moscow Olympiad 1978]


First, we introduce some basic knowledge of vector operations.
Let u = [a. b] and v = Im, nJ be two vectors.
Define their dot product u v = am + bn.
It is easy to check that

(i) vv = m2+n2 = 1v12,

that is, the dot product of vector with itself

is the square of the magnitude of v and v v 0 with equality if


and only if v = [0.0];

(ii) u v = v u;

(iii) u (v + w) = u v +

w. where w is a vector;

(iv) (cu) v = c(u . v), where c is a scalar.


.

When vectors u and v are placed tail-by-tail at the origin 0. let A and
.8 be the tips of u and v. respectively. Then
= v u.
Let LAOB =0.
Applying the law of cosines to triangle AOB yields

= AB2
= 0A2-I-0B2-20A.OBcosO
=

juj2 + v]2

21u!JvJ cosO.

4. Solutions to Advanced Problem

90

It follows that
(v u) (v
.

u) =

uu+v

v 2IuIlvIcosO,

or

cosO=
Consequently, if 0

IuHvI

900, u v 0.

Consider vectors v1 = [ai, a2], v2 =

[a3,

a4], v3 =

[a5,

a6], and v4

[a7, as].

Note that the numbers a1a3+a2a4. a1a5+a2a6, a1a7+a2a8, a3a5+a4a6,


a3a7 + a4a8, a5a7 + a6a8 are all the dot products of distinct vectors Vj
and
Since there are four vectors, when placed tail-by-tail at the origin, at
least two of them form a non-obtuse angle, which in turn implies the
desired result.

Problem 76 [IMO 1996 short list]


Let a, b and c be positive real numbers such that abc =
Prove that
ab

ca

bc

1.

a5+b5+ab+b5+c5+bc+c5+a5+ca 1.
Solution 76
We have
a5 + b5

a2b2(a + b),

because
(a3 b3)(a2

b2)

0,

with equality if and only if a = b. Hence


ab

ab

a2b2(a+b)+ab
1

ab(a+b)+1

ab(a+b+c)

abc
C

a+b+c

Likewise,
bc

b5 + c5 + bc

a
a+b+c

45olutions to Advanced Problems

91

and

+ a5 + ca a + b + c
the last three inequalities leads to the desired result.
Equality holds if and only if a = b = c = 1.
c5

Comment:

Please compare the solution to this problem with the

second solution of problem 13 in this chapter.

Problem 77 [Czech-Slovak match 1997]


Find all functions f R * R such that the equality

f(f(x)+y) =f(x2 y) +4f(x)y


holds

for all pairs of real numbers (x, y).

Solution

77

Clearly. f(x) = x2 satisfies the functional equation.


Now assume that there is a nonzero value a such that f(a)
x2 1(x)

Let y =

a2.

in the functional equation to find that

(f(x)+x2)

(f(x)+x2)

=
2f(x)(x2
In

1(x)

or 1(x) = x2.

both cases. f(0) = 0.

Setting x = a, it

follows from above that either f(a)

0 or 1(a) = 0

or

f(a)=a2.

The latter is false, so f(a)

Now,

let x = 0 and then x = a

1(u)

and

Let

in the

= f(y),

functional equation to find that

1(y) = f(a2

so

1(n)

that

0.

= f(y) = f(a2 + y);

is, the function is periodic with nonzero period a2.

y=

a2

in the original functional equation to obtain

f(f(x)) = 1(1(x) + a2) =


However, putting y =
for all x.

in

f(x2 a2)

+ 4a2f(x) = f(x2) + 4a2f(x).

the functional equation gives 1(1(x))

= 1(x2)

4. Solutions to Advanced Problems

92

Thus, 4a2f(x) = 0 for all x. Since a is nonzero, 1(x) =


Therefore, either 1(x) = x2 or f(x) = 0

for all x.

Problem 78 [Kvantj
Solve the system of equations:
3x y
x2 + y2

x+

=3

x+3y =0.

x2 + y2

Solution 78, Alternative 1


Multiplying the second equation by i and adding it to the first equation
yields

(3xy)(x+3y)i
or

i(xyi)
x2+y2

x2+y2
Let z = x + yi. Then

=3,

!_ xyi
z x2 + y2

Thus the last equation becomes

z+

3i
z

or

z23z+(3i) =0.
Hence

3v'34-4i

3(1+2i)

that is, (x,y) = (2,1) or (x,y) = (1,i).

Solution 78, Alternative 2


Multiplying the first equation by y, the second by x, and adding up yields
2xy +
or 2xy

3y.

(3x

y)y

(x + 3y)x
x2 + y2

It follows that y

0 and
3y + I

45olutions to Advanced Problems

this into the second equation of the given system gives

[(3y+ 1)2

(3Y+ 1)

= o.

or

4y4 3y2

it follows that y2

0.

1 and that the solutions to the system are (2, 1) and

(1,i).

problem 79 [China 1995]


Mr. Fat and Mr. Taf play a game with a polynomial of degree at least 4:

+ ... +_x + i.
They fill in real numbers to empty spaces in turn.
If the resulting polynomial has no real root. Mr. Fat wins; otherwise, Mr.
Taf wins.

If Mr. Fat goes first, who has a winning strategy?

Solution 79
Mr. Taf has a winning strategy.
We say a blank space is odd (even) if it is the coefficient of an odd (even)
power of x.

First Mr. Taf will fill in arbitrary real numbers into one of the remaining
even spaces, if there are any.

Since there are only n


Space left after 2n

1 even spaces, there will be at least one odd


plays, that is, the given polynomial becomes

p(x) = q(x)
where

+_x2t_1,

s and 2t I are distinct positive integers and q(x) is a fixed

Polynomial.

We claim that there is a real number a such that

p(x) = q(x) +
Will

+_x2t1

always have a real root regardless of the coefficient of

Then

Mr. Taf can simply fill in a in front of x8 and win the game.

4. Solutions to Advanced
Now we prove our claim. Let b be the coefficient of x2t_1 in p(x). Note

that
+

p(l)
+ 2s_2t+la + b) + [q(1) + (1)8a

Since s

+ q(_1)) +

2t 1 2s2t+1

+ (_1)8

o.

Thus
1

a=
is

+ q(1)

2s2t+1 + (_1)8

well defined such that a is independent of b and

+p(l) = 0.
It follows that either p(l) = p(2) = 0 or p(l.) and p(2) have different
signs, which implies that there is a real root of p(x) in between 1 and
2.

In either case, p(x) has a real root regardless of the coefficient of


as claimed.
Our proof is thus complete.

Problem 80 [IMO 1997 short list}


Find all positive integers k for which the following statement is true:
F(x) is a polynomial with integer coefficients satisfying the condition
for

if

c=0,1

then F(O) = F(1) = ... = F(k + 1).

Solution 80
The statement is true if and only if /c 4.
We start by proving that it does hold for each Ic 4.
Consider any polynomial F(x) with integer coefficients satisfying the
inequality 0 F(c) Ic for each c e {0. 1
k + 1}.
Note first that F(k + 1) = F(0), since F(k + 1) F(0) is a multiple of
Ic + 1 not exceeding Ic in absolute value.
Hence
F(x) F(0) = x(x

Ic

1)G(x).

45olutions to Advanced Problems


G(x) is a polynomial with integer coefficients.
consequently,
where

k jF(c) F(O)j = c(k + I c)IC(c)I


for each c E

k}.

The equality c(k + 1 c) > k holds for each cC {2, 3


equivalent to (c 1)(k c) > 0.

k 1},

as it is

that the set f2. 3


k 1} is not empty if k 3, and for any c in
this set. (1) implies that
< 1.
Since C(c) is an integer, C(c) 0.
Note

Thus

F(x)F(0)=x(x2)(x3)''(xk+1)(xk1)H(x).

(2)

where H(x) is a polynomial with integer coefficients.

To complete the proof of our claim, it remains to show that 11(1) =


H(k) = 0.
Note that for c = 1 and c = k, (2) implies that
Ic

IF(c) F(0)I = (Ic 2)!

Ic

IH(c)I.

Fork4,(k2)!>1.
Hence H(c) = 0.
We established that the statement in the question holds for any k 4.
But the proof also provides information for the smaller values of Ic as
well.

More exactly, if F(x) satisfies the given condition then 0 and k + 1 are
roots of F(x) and F(0) for any Ic
and if Ic 3 then 2 must also be
a root of F(x) F(0).
Taking this into account, it is not hard to find the following counterexamples:

F(x)=x(2x)

fork=1.

for Ic = 2.
F(x) = x(3 x)
F(x)=x(4x)(x2)2 for k=3.

4. Solutions to Advanced Problems

96

Problem 81 [Korean Mathematics Competition 2001J


The Fibonacci sequence

is given by

(n.EN).
Prove

that

r'3

i'3

L 2n+2

2n2

for all n 2.

Solution 81
Note

that

whence
for

=0

(1)

all n 2.

Setting a =

and c =

b=

in the algebraic identity


abbcca)

gives
rv,n3

htf2n

r,3

n3

Applying (1) twice gives

The desired result follows from


tlr2n+2r2nr2n_2

r'3 r'

r'

r'

r'2

r2n)\

4. Solutions to Advanced Problems

97

problem 82 [Romania 1998]


Find all functions u : R * R for which there exists a strictly monotonic
function f : R * R such that

f(x +

= f(x)u(y) + f(y)

y)

for all x,y ER.

Solution 82
The solutions are u(x) = ax. a E R.
To see that these work, take f(x) = x for a =

1.

1, take 1(x) =a 1; then


f(x + y) =

(ax

+ f(y)

for all x, y E R.
Now suppose u R *

R are functions for which f is strictly


R, f R
monotonic and f(x + y) = f(x)u(y) + f(y) for all x, y E R.
We must show that u is of the form u(x) = ax for some a E R+. First,
letting y = 0, we obtain f(x) = f(x)u(0) + f(0) for all x E R.
1 would imply f(x) = f(O)/(1 u(O)) for all x, which
Thus, n(O)
would contradict the fact that f is strictly monotonic, so we must have
u(0) = 1 and f(0) = 0.
Then f(x) 0 for all x 0.

Next, we have

f(x)u(y) + f(y) = f(x + y) = f(x) + f(y)u(x),


or

f(x)(u(y) 1) = f(y)(u(x)

1)

for all x,y ER. That is,

for all xy

u(x) I

f(x)

u(y)

1(y)

0.

It follows that there exists C E R such that


u(x) I

f(x)
for all x

0.

Thus, u(x) = 1+Cf(x) for x


also holds for x = 0.

0: sinceu(0) =

1,

f(0) = 0, this equation

4. Solutions to Advanced Problem

98

If C = 0, then u(x) = 1 for all x, and we are done.


Otherwise, observe

u(x+y) = 1+Cf(x+y)
+ Cf(x)u(y) + Cf(y)
= u(y)+Cf(th)u(y)
=

u(x)u(y)

for all x,y cR.


Thus u(nx) = u(x) for all n Z, x R.
Since u(x) = 1 + Cf(x) for all x, u is strictly monotonic, and u(x) =
1/u(x) for all x, so u(x) >

Let a = u(1) >

0;

for all x as u(O) = 1.


then u(n) = a for all n N, and
0

u(p/q) = (u(p))lk =
for

allpe Z, q E N, so u(x) =

ax

for allxE Q.

monotonic and the rationals are dense in


for all x E R.
Thus all solutions are of the form u(x) = ax, a E
Since u

is

we have u(x) =

ax

Problem 83 [China 1986]


be complex numbers such that
Let z1 Z2,
,

Prove that there exists a subset S of {zi,

Z2,... ,

such

that

zS

Solution 83, Alternative 1


Let
2, and 3 be three rays from origin that form angles of 60, 180,
and 300, respectively, with the positive
(here 4 =
For i = 1,2,3, let 7Z, denote the region between and
Then
including the ray

1=

jZkj+ >

ZkH

!ZkI
ZkEl?..3

By the Pigeonhole Principle, at least one of the above sums is not less

than 1/3.

4. Solutions to Advanced Problems

(otherwise, we apply a rotation, which does not effect the


it's
magnitude of a complex number), Let zk = xk + 1Yk. Then for Zk e 7Z3,
Say

XkI

Xk

ZkI/2.

>
as

desired.

Solution 83; Alternative 2


We prove a stronger statement: there is subset S of {zi, z2, .. .
that

z,.,}

such

zES

For

I k

n, let Zk
1

Xk +

iyk. Then

=
XkO

Xk<O

>

yk<O

By the Pigeonhole Principle, at least one of the above sums is not less

than 1/4. By symmetry, we may assume that

Xk>O

XkO

Consequently,

XkO

Comment:
Using advanced mathematics, the lower bound can be
further improved to 1/ir.

Problem 84 [Czech-Slovak Match 1998]


A polynomial P(x) of degree n 5 with integer coefficients and n distinct
integer roots is given.
Find all integer roots of P(P(x)) given that 0 is a root of P(x).

4. Solutions to Advanced Problems

100

Solution 84
The roots of P(x) are clearly integer roots of P(P(x)); we claim there
are no other integer roots.
We prove our claim by contradiction. Suppose, on the contrary, that
P(P(k)) = 0 for some integer k such that P(k) 0.
Let

P(x) =a(xri)(x r2)(xr3).. .(x rn),


where a.rl,r2,...,rfl are integers,

Since P(k)

Since the

we must have 1k
1 for all i.
are all distinct, at most two of 1k r2j. k
0,

equal

r31, 1k

1, so

Ia(kr2).(krn_i)I
IP(k)l

Also note that P(k) =


for some i0, so P(k)!
Now we consider the following two cases:
and

1.

Then P(k)I

rn!.

1k!

> rn!, a contradic-

rn)!

tion.
2.

1k!

< rn!, that is,


b. For

xe

[a,

1k!

b}, the

1.

Let a. b, c be real numbers,

function

f(x) =x(cx)
reaches its minimum value at an endpoint x =

or x =

b,

or at

both endpoints.
Thus
k(k

rn)I = k!!rn kI !k!(!Tn!

1.

1k!)

It follows that
rn!

P(k)!

1),

which implies that Tn! K 2. Since n 5, this is only possible if

P(x) = (x + 2)(x + 1)x(x 1)(x


But then it is impossible to have k
diction.

2).

and 1k! rn!, a contra-

Solutions to Advanced Problems

101

our assumption was incorrect, and the integer roots of P(P(x)) are
exactlY all the integer roots of P(x).
Thus

problem 85 [Belarus 1999}


and

Two real sequences x1, x2

are defined in the following

Y2

way.

and

yn
1

for all n 1. Prove that 2 < XnYn <3 for all n> 1.

Solution 85, Alternative 1


Let

and note that the recursion for

is equivalent to

Zn+1 = Zn +

= x1; since the


and
Also note that Z2 =
recursion, this means that Zn = Xn_1 for all n> 1.

satisfy the same

Thus,
XnYn =

Xn

=
2n

Xn
.
Xn_1

Note that
>

Thus
2Xn_i and XnYn > 2, which is the lower bound of the desired
inequality.

Since Xn5 are increasing for n> 1, we have

= 3>
which implies that
2Xn_i >

Thus 3Xn_i > Xn. which leads to the upper bound of the desired inequality.

Solution 85, Alternative 2


Setting

= cot 0,, for 0 < 0,, <900 yields

= cot

+ cot2

= cot

= cot

4. Solutions to Advanced Problen

102

Since

300

30, we have in general

Similar calculation shows

that
tan 2
2

tan

tan

0. tan2

And since for n>

is positive and XnYn > 2.

< 30.

we have

<

tan2

so that XnYn

we also hare

<3.

Comment: From the closed

forms

for

and

in the second solution,

we can see the relationship

yn =
used

xfll

in the first solution.

Problem 86 [China 1995]


For a polynomial P(x), define the difference of P(x) on the interval 4 b)

({a,b), (a,b), (a,bJ) as P(b) P(a).


Prove that it is possible to dissect the interval 9. 1] into a finite number
of

intervals and color them red and blue alternately such that, for every

the total difference


equal to that of P(x) on blue intervals.
What about cubic polynomials?
quadratic polynomial P(x),

of P(x)

on

red intervals

is

Solution 86
For

an interval i,

let

denote

the difference of polynomial P

on

i.

For a positive real number c and a set S R, let S + c denote the set
obtained by shifting S in the positive direction by c.
We

prove a more general result.

Lemma
Let be a positive real number, and let k be a positive integer. It is
always possible to dissect interval Ik

[0.

into

a finite number of

such that, for every


polynomial P(x) with degP k, the total difference of P(x) on the red
intervals is equal to that on the blue intervals.
intervals and color them red and blue alternatively

45olutions to Advanced Problems

103

proof

We induct on k.

For k = 1, we can just use intervals


and (t.2t]. It is easy to see
that a linear or constant polynomial has the same difference on the two
intervals.

Suppose that the statement is true for /c = n, where n is a positive


integer; that is. there exists a set
of red disjoint intervals and a set
fl
of blue disjoint intervals such that
U
=
and,
= @.
for any polynomials P(x) with deg P n. the total differences of P on
is equal to that of P on
consider polynomial f(x) with deg f n + 1. Define

g(x) = f(x +
Then deg h

and h(x) = f(x) g(x).

n. By the induction hypothesis,

=
or

Arf+

It follows that

Arf,
where
11-n+1

and

D L)ii(D

)flD\

and
both contain the number
that number may be
removed from one of them.)
It is clear that
and
form a dissection of 'n+l and, for any
(If

Polynomial f with degf


equal to that of f on

n + 1, the total difference of f on

The only possible trouble left is that the colors in


be alternating (which can happen at the end of the

is

might not
and the beginning

But note that if intervals i1 = [a1, b1 J and i2 = [b1, c1 } are in the same
Color, then

+
Wherej3 = [ai,ci].

4. Solutions to Advanced Problems

104

we can simply put consecutive same color intervals


into one bigger interval of the same color.
Thus, there exists a dissection
Thus. in

=
such

that, for every polynomial f(x) with deg f n + 1.

This completes the induction and the proof of the lemma.

Setting first =

and then =

in the lemma, it is clear that the

answer to each of the given questions is

Problem 87 [USSR 1990]


Given a cubic equation
S3

+ _X2 + _X + = 0,

Mr. Fat and Mr. Taf are playing the following game.
In one move, Mr. Fat chooses a real number ana Mr. Taf puts it in one
of the empty spaces.
After three moves the game is over.
Mr. Fat wins the game if the final equation has three distinct integer
roots.

Who has a winning strategy?

Solution 87
Mr. Fat has a winning strategy.
Let the polynomial be x3 + ax2 + bx + c. Mr. Fat can pick 0 first. We
consider the following cases:
(a) Mr. Taf chooses a = 0, yielding the polynomial equation
x3 + bx + c = 0.

Mr. Fat then picks the number (inrip)2, where in. n, and p are
three positive integers such that
m2 + n2 =

If Mr. Taf chooses b = (n2rip)2, then Mr. Fat will choose c = 0.


The given polynomial becomes
x(x n2np)(x + rnnp).

4golutions to Advanced Problems

If Mr. Taf chooses c =

(rnnp)2. then Mr. Fat will choose


fl2p2

b = ?TL2fl2

p27772.

The given polynomial becomes

(x + rn2)(x + n2)(x

p2).

(b) Mr. Taf chooses b = 0. yielding the equation


x3 + ax2

0.

Mr. Fat then picks the number


m2(m + 1)2(rn2 -+ m +

where rn is an integer greater than 1.


If Mr. Taf chooses

a = rn2(m + 1)2(m2 + m +
then Mr. Fat can choose
c = m8(in + 1)8(in2 + rn + 1)6.
The polynomial becomes
(x

rnp)[x

(rn

1)pJ[x

+ m(rn + 1)pJ,

where
p = m2(n2 + 1)2(rn2 + rn + 1)2.

If Mr. Taf chooses

c=m2(m+ 1)2(m2 +m+ j)3,


then Mr. Fat can choose

a=

(m2

+m+

1)2.

The polynomial becomes

(x+mq)[x(rn+1)qffxm(m+
where

q = in2 + in +

1.

4. Solutions to Advanced Problems

106
(c)

Mr. Taf chooses c =

0.

Then the problem reduces to problem 40 of the previous chapter.


Mr. Fat needs only to pick two integers a and b such that
ab(a 1)(b1)

and a+b= 1.
The polynomial becomes either x(x 1)(x a) or x(x 1)(x

b).

Our proof is complete.


Below is an example of what Mr. Fat and Mr. Taf could do:
F

3600

4.9.73

0
481

Roots
60,0.60

.38.76

16, 9,25

8.27.49,
4. 27 . 49,

89.
c
"

"

"

49

14, 21,42
3,0,1

0,1.2

Problem 88 [Romania 1996]


Let n> 2 be an integer and let f : R2
any regular n-gon A1A2. .

49

be a function such that for

f is the zero function.


Solution 88
We identify R2 with the complex plane and let c =
Then the condition is that for any z C C and any positive real t,

In particular, for each of k

1,. ..

, n,

we obtain

4. Solutions to Advanced Problems

107

we have

gumming over

,n=1 k=1

Form = n the inner sum is nf(z); for other m, the inner sum again runs
over a regular polygon, hence is 0.
Thus f(z) = 0 for all z E C.

Problem 89 [IMO 1997 short list]


Let p be a prime number and let f(s) be a polynomial of degree d with
integer coefficients such that:

(i) 1(0) = 0, f(1) =

1;

(ii) for every positive integer n, the remainder upon division of 1(n)
by p is either 0 or 1.

Prove that dp 1.
Solution 89, Alternative 1
For the sake of the contradiction, assume that d

p 2.

Then by Lagrange's interpolation formula the polynomial f(s) is


determined by its values at 0, 1,
1 (x)

., p

2;

that is,

k=O

p-2

Setting x = p

k)5

k=O
1

f(p

(1) .. (k

xk

(x

k!(_1)P-k

p +2)

(x

P+2)

(p k 2)!

gives
1)

=
=

>f(k)

k)

(_1)Pkk!
p2

(modp).
It follows that

S(f) := f(o) + f(1) + ... + f(p -1)

(mod p)

(1)

4. Solutions to Advanced Problems

108

other hand, (ii) implies that S(f) j (mod p), where j


the number of those Ic E
1
1
p 1} for which 1(k)
(mod p)
But (i) implies that 1 j < p 1.
So S(f) 0 (mod p), which contradicts (1).
Thus our original assumption was wrong. and our proof is complete.
On the

Solution 89, Alternative 2


Again, we approach the problem indirectly.
Assume that d p 2, and let

1(x) =

+ a1x + a0.

Then
pi

S(f) =

pi p2

p2

1(k) =

p2

=
i=0

k=O i=0

k=0

pi

=
k=O

i=0

pi

where

(modp)foralli=0,1

p2.

We use strong induction on I to prove our claim.


The statement is true for i 0 as So = p.
Now suppose that So S1
0 (mod p) for some 1 i
..
p2. Note that
pi

pI

k=Oj=0
(I

j0

(mod p)

Since 0 < i + 1 <p, it follows that


0 (mod p). This completes the
induction and the proof of the claim. Therefore,

S(f)

(mod p).

=
The rest is the same as in the first solution.

4Solutions to Advanced Problems

109

problem 90
Let n be a given positive integer.
with a0 =

Consider the sequence a0, a1.

ak = ak_I
for k = 1, 2,
Prove that

and

+
n
1

n.
1

< 1.

1
Ti

Solution 90, Alternative 1


We prove a stronger statement: For k = 1.

2
71

(1)

We use induction to prove both inequalities.


We first prove the upper bound, For k = 1, it is easy to check that
1

2n+1

4n

4n

2ni

a1 = + =
Suppose that
<

2n k'
for some positive integer k <n. Then
ak

= (n-f-ak)
<

271_k(Ti+271_k)

n(2nk+1)

(2nk)2

<

2nk1'

(2n k+ 1)(2n ki) = (2nk)2 1< (2n k)2


Thus our induction step is complete. In particular, for k = n

afl=ak+1<

n
2n (n

1)

=1,

1,

4. Solutions to Advanced Problems

110
as

desired.

Now we prove the upper bound. For k =


=

2n+1

1,

it is easy to check that

n+1

4n

Suppose that

n+1
2ri k + 2'

ak>

for some positive integer k < n. Then


ak+l ak +

(n -I- 1)2

ri + 1

> 2n_k+2+n(2n_k+2)2

It follows that

n+1

n+1

ak+i_2k+l

(ri.l-1)2

(2n_k+l)(2n_k+2)+n(2n_k+2)2
n+1

"ri+l

n+1

/1

2nk+2)2

2nk+2\

This

complete the induction step. In particular, for n

n+1
2n(n1)+1
as

2n_k+1) >0.
=

1,

72+1

n+2

n+2

ri

desired.

Solution

90, Alternative 2

Rewriting the given condition as


Ti

ak

ak

we obtain

rt+ak_1

It is clear that aks are increasing.


Thus
I

4. Solutions to Advanced Problems

111

Thus (2) implies that


1
<
n
ak_i
1

ak

for k = 1, 2, . .. , fl
Telescoping summation gives
1

1
<1

a0
or
1
>

that is,

1 = 21 =
a0

<1, which gives the desired upper bound.

< 1, and, since aks are increasing,

Since

1,

< 1 for

<

k=1,2,...,n.
Then (2) implies
1

ak

n+ak_1

>

n+1

fork=1,2,...,ri.
Telescoping sum gives
1
1
-->
a0
n+1

or

1<1

n+ln+1

ao

that is,

n+2

72

n+1 =1 1 >1,
n
n+2
n-f-2

which is the desired lower bound.

Problem 91 [IMO 1996 short list]


Let a1, a2,..

be nonnegative real numbers. not all zero.

(a) Prove that

positive real root R.


(b) Let A =

and B =

Prove that AA RB.

..

jaj.

has precisely one

4. Solutions to Advanced Problem

112

Solution 91
(a) Consider the function
a1

a2

Note that f decreases from oo to 0 as x increases from 0 to oc.

Hence there is'a unique real number R such that f(R) = 1, that is,
there exists a unique positive real root R of the given polynomial.
(b) Let c3 = ad/A.
Then c3s are non-negative and

c2

1.

Since ln x is a convex function on the interval (0, oo), by Jensen's

inequality,

Ecj

=-In(f(R))=0.

ln

ft follows that

c3(lnA+jlnR) 0
j=1
or

c3

= a3 /A, we obtain the desired inequality.

Comment: Please compare the solution of (a) with that of the problem
15 in the last chapter.

Problem 92
Prove that there exists a polynomial P(x, y) with real coefficients such
that P(x, y) 0 for all real numbers x and y, which cannot be written
as the sum of squares of polynomials with real coefficients.

Solution 92
We claim that

P(x,y) =

(x2 +y2 1)x2y2

is a polynomial satisfying the given conditions.

First we prove that P(x, y) 0 for all real numbers x and y.

4. Solutions to Advanced Problems

113

If x2 + y2 I 0, then it is clear that P(x,y) > 0; if x2 + y2 1 <0,


then applying the AM-GM inequality gives

or

(x2 + y2 l)x2y2

It follows that P(x,y) 0.


We are left to prove that P(x, y) cannot be written as the sum of squares
of polynomials with real coefficients.
For the sake of contradiction, assume that

P(x,y) =
Since

degP =

3.

6,

Thus

=
+

+ C2y2 +

y)2, of terms x6

Comparing the coefficients, in P(x, y) and


and y6 gives

=0,

orA' =D2=Oforalli.
Then, comparing those of x4 and y4 gives

Next, comparing those of x2 and y2 gives

or H, =

0,

for all

Thus,

+ I2y +

+ F,.xy +

4. Solutions to Advanced Problems

114

But, finally, comparing the coefficients of the term x2y2, we have

= -1,
which is impossible for real numbers
Thus our assumption is wrong, and our proof is complete.

Problem 93 [IMO 1996 short list]


For each positive integer n, show that there exists a positive integer k
such that
k = f(x)(x + 1)2n +
g

with integer coefficients, and find the smallest

such /c as a function of n.

Solution 93
First we show that such a k exists.
Note that x + 1 divides 1
Then for some polynomial a(s) with
integer coefficients, we have

(1 + x)a(x) = 1

= 2 (1 +

or

2=(1+x)a(x)+(1 +x272).
Raising both sides to the (2n)th power, we obtain

= (1 +

+ (1 +

where b(s) is a polynomial with integer coefficients.


This shows that a k satisfying the condition of the problem exists. Let
/c0 be the minimum such k.
Let 2n = 2r . q, where r is a positive integer and q is an odd integer.
=
We claim that

First we prove that 2q divides k0. Let t =


(5t + 1)Q(x), where

Q(x) = 5t(q_l)
The roots of

Note that

+ ...

+ 1.

+ 1 are

((2m1)ir\

f(2m1)ir\

),

m=1,2,...,t;

Solutions to Advanced Problems


that is,

Let

f(s)

115

R(x)=xt+l=(x_wi)(x_w2)..(x_wt).
and

g(x) be polynomials with integer


k0

coefficients

f(x)(x+1)272+g(x)(x272+1)

f(x)(x + 1)2n + g(x)Q(x)(xt + 1).

such that

It follows that
f(Wm)(Wm+1)Th=ko,

(1)

Since r is positive, t is even. So

2=R(1) =

+w2)(1+wt).

is a symmetric polynomial in

Since

..

with integer coefficients, it can be expressed as a polynomial with integer

coefficients in the elementary symmetric functions in w1, w2,.


and

..

therefore

F = f(wl)f(w2)

is an integer.

Taking the product over m = 1,2,... , t, (1) gives 2272F = ku or

It follows that 2q divides k0.


It now suffices to prove that k0
Note that Q(1) = 1.
It follows that
Q(x) =(x+1)c(x)+1,
where

is a polynomial with integer coefficients.

c(s)

Hence

(x +

1)2n

= Q(x)d(x) + 1,

for some polynomial d(x) with integer coefficients.


Also observe that, for any fixed m,

Thus
(1 +(iJm)(1

and

writing

(1 +w2t_1)

= R(1) = 2,

(2)

4. Solutions to Advanced Problems

116
we

find that for some polynomial h(x), independent of m. with integer


such that

coefficients

(1 + w)th(w) = 2.
But then (x + 1)h(x)

is divisible by xt + 1 and hence we can write

(x + 1)h(x) = 2+ (xt + 1)u(x),


for some polynomial u(x) with integer coefficients.
Raising both sides to the power q we obtain

= 2q + (xt + 1)v(x),

(x +

(3)

where v(x) is a polynomial with integer coefficients.


Using (2) and (3) we obtain

(x +

+ 1)v(x)
= Q(x)d(x)(xt + 1)v(x) + (xt + 1)v(x)
= Q(x)d(x)(xt -I- 1)v(x) + (x +

2q,

that is,
2q

where

(x)

= fi(x)(x

+ g1(x)

+ 1),

and 91(x) are polynomials with integer coefficients.

Hence k0 < 2q, as desired.

Our proof is thus complete.

Problem 94 [USAMO 1998 proposal, Kiran Kedlaya]


Let x be a positive real number.

(a) Prove that

(ni)!

_!

(b) Prove that

(ni)!

4. Solutions to Advanced Problems

117

Solution 94
We use infinite telescoping sums to solve the problem.
(a) Equivalently, we have to show that
1

that

n!x

(ni)!

n!

and this telescoping summation yields the desired result.


(b) Let

(n-i)!

f( )X

Then,

by (a), 1(x)

In particular, 1(x)

write

f as

converges

to 0 as x approaches oo, so we can

an infinite telescoping series

+k

-1) - f(x + k)].

On the other hand, the result in (a) gives

f(x-1)-f(x)

(ni)!

x1(x+1)(x+n)

(1)

4. Solutions to Advanced Problems

118

Substituting the la.st equation to (1) gives

as desired.

Problem 95 [Romania 1996]


Let n 3 be an integer, and let

XcS={1,2,,,.,n3}
be a set of 3n2 elements.
Prove that one can find nine distinct numbers
such that the system

aix+biy+ciz =
a2x+b2y+c2z =
a3x+b3y+c3z =

(i = 1,2,3) in X
0

0
0

has a solution (xo, Yo, zo) in nonzero integers.

Solution 95
Label the elements of X in increasing order Xi

<<

and put

X1 = {x1,...
X2
X3
Define

=
=

the function f : X1 x X2 x X3
f(a, b, c) =

(b

x S as follows:

a, c b).

The domain of f contains n6 elements,


The range of f, on the other hand, is contained in the subset of S x S
of pairs whose sum is at most n3, a set of cardinality
1

By the Pigeonhole Principle, some three triples (a2,


map to the same pair, in which case x =
= ci
is a solution in nonzero integers.

cj) (i = 1,2,3)
a1,z = a1 bi

4. Solutions to Advanced Problems


Note

ai

119

that
cannot equal
since X1 and X2 are disjoint, and that
a2 implies that the triples (ai,bi,c1) and (a2,b2,c2) are identical,

a contradiction.
Hence the nine numbers chosen are indeed distinct.

problem 96 [Xuanguo Huang]


Let n 3 be an integer and let x1, x2,

be positive real numbers.

Suppose that
= 1.

Prove that

(1

Solution 96
By symmetry, we may assume that x1 x2
following lemma.

Lemma
1

Prooft

+ x3

Since n 3, and, since

1,

we have

or

>

It follows that x2x3> 1. Thus

1+x2

1+x3

(1+x1)(1+x3)

(1+x2)(1+x3)

0,

We have the

4. Solutions to Advanced Problems

120
as

desired.

By the lemma, we have

>

1+xi

1+x2

and, since

it follows by the Chebyshev Inequality


1

(1

By

the Cauchy-Schwartz Inequality, we have


2

or

(2)

Multiplying by

on both sides of (2) and applying (1) gives

which in turn implies the desired inequality.

4. Solutions to Advanced Problems

121

problem 97
be distinct real numbers.
Define the polynomials
Let X1,X2,. .

P(x) =

/1

and

Q(x)=P(x)(

\XXj

XX2

+..,+

Yn1 be the roots of Q.

Let Y1,Y2

Show that
minlx,,x31 <minly2y71.

Solution 97
By symmetry, we may assume that

d= minly, Yjl=
Let 8k =

Xk, for

k = 1.2,..., n.

By symmetry, we may also assume that .s1 <

<

i.e., x1 >

For the sake of contradiction, assume that

= minx,

d minlx2

= mins3

(1)

i(j

Since P has no double roots, it shares none with Q.


Then
1

(\Y,X1

yjX2

= Q(Yi)

0,

or
1

YiX1

Y%X2

=0.

In particular, setting i = 1 and i = 2 gives


(2)

We

claim that there is a k such that sk(sk + d) <0, otherwise, we have


1

Sk+d

8k

4. Solutions to Advanced Problems

122
for. all

k, which in turn implies that

which contradicts (2).


< 0 < Sk+d.
Let j be the number of ks such that sk(sk+d) < 0, that is, Sk
the
same
sign. In
have
d
and
+
A simple but critical fact is that 8k
fact, suppose that

then
> 0 if and only if /c > i + 1, that is Sk + d> 0.
Then
From (1), we obtain Sk + d Sk+3, and, since sk + d and 3k+3 have the
same sign, we obtain
1

Sk

for all k =

,n

>1

+ d 8k+j

j. Therefore,

or
(3)

Also note that

E!<o<E 1
+
k=1

Sk

Adding (3) and (4) yields


Ti

fl

which contradicts (2).

Thus our assumption is wrong and our proof is

4. Solutions to Advanced Problems

123

Problem 98 [Romania 1998]


Show that for any positive integer n, the polynomial

f(x)=(x2+x)2"+l
cannot be written as the product of two non-constant polynomials with
integer coefficients.

Solution 98
Note that 1(x) = g(h(x)), where h(x) =

x2

+ x and 9(y) = y2" + 1.

Since

and
is even for
criterion.

1 k

1,

g is irreducible, by Eisenstein's

Now let p be a non-constant factor of f, and let r be a root of p.


Then g(h(r)) = f(r) = 0, so .s h(r) is a root of g.
Since s = r2 -f- r C Q(r), we have Q(s) C Q(r), so

deg p deg(Q(r)/Q) deg(Q(s)/Q) =

deg

g=

Thus every factor of f has degree at least T'.


Therefore, if f is reducible, we can write f(x) = A(x)B(x) where A and
B have degree

Since
have

+5

+x+1) 2"

(mod2).

+ x + 1 is irreducible in Z2[x], by unique factorization we must

A(s)

B(s)

+ x + 1)2"'

(mod 2).

Thus, if we write
A(s)

B(s) =

+ ao,
+
+ .. + b0,

b0 are odd and all the other coefficients


then
ao,
are even. Since f is monic, we may assume without loss of generality

4. Solutions to Advanced Problems

124

= 1; also, a0bo = f(O) =


=
that
no real roots, so a0 = b0 = 1.

1.

but a0 > 0,

> 0 as f has

Therefore,
([x2n+2"1] + [x2"1])(g(x)h(x))

( i=2"1
+

+
EQ
as
But

+
(mod 4)

is even.

+ [x2"'])(f(x))

is odd by Lucas's theorem, so

and

([x2"+2"'] + [x2"')) (1(x))

(mod 4),

a contradiction.
Hence f is irreducible.

Problem 99 [Iran 1998]


Let Ii, 12,13 R R be functions such that
aifi + a212 + a3f3
is monotonic for all a1, a2, a3 E
Prove that there exist Ci, c2, c3 c R, not all zero, such that

cifi(x) + c2f2(x) + c3f3(x) =

for all x E R.

Solution 99, Alternative 1


We establish the following lemma.

Lemma: Let f, g R p IR be functions such that f is nonconstant and


af + bg is monotonic for all a, b c R. Then there exists c R such that
g cf is a constant function.
Proofi

Let s, t be two real numbers such that f(s)

1(t).

4. Solutions to Advanced Problems


Let

125

g(s)g(t)
f(s) - f(t)

Let h1 = g d1f for some d1 E R.


Then h1 is monotonic. But
hi(s) h1(t)

Since f(s)

= g(s) g(t)

f(t)

d1(f(s)

f(t)) = (f(s)

f(t))(u d1).

0 is fixed, the monotonicity of h1 depends only on

thesignofud1.
Since f is nonconstant, there exist xl,x2 c

such that 1(x1)

f(x2).

Let

c= g(xl)g(x2)
f(xi)

f(x2)

= g cf.
Then r = h(xi) = h(x2) and the monotonicity of h1 =

and h

d1f, for each

d1, depends only on the sign of c d1.


We claim that h = g cf is a constant function.
We prove our claim by contradiction.

Suppose, on the contrary, that there exists x3 c R such that h(xs) r.


and f(x2) f(xi) is
Since f(xi) f(x2), at least one of f(xi)
true.
Without loss of generality, suppose that 1(x1)
Let

c' = g(x1)

f(x3).

g(x3)

f(xi) f(x3)

Then the monotonicity of h1 also depends only on the sign of c' d1.

Since h(x3)

r = h(xi),
f(xi)

hence

cd1

f(x3)

=c';

c' d1.

So there exists some d1 such that h1 is both strictly increasing and decreasing, which is impossible.
Therefore our assumption is false and h is a constant function.

Now we prove our main result.


If fi f2, 13 are all constant functions, the result is trivial.
Without loss of generality, suppose that Ii is nonconstant.

4. Solutions to Advanced Problems

126

For a3 = 0, we apply the lemma to Ii and 12, so 12 = cf1 +d; for a2

we apply the lemma to fi and 13, so


Here c, c', d, d' are constant.

0,

= c'f1 + d'.

We have

(c'd cd')f1 + d'f2 df3 = (c'd cd')fi + d'(cfi + d) d(c'fi + d') =

If (c'dcd',d',d)

0.

(0,0,0), then let


(ci, c2, c3) = (c'd cd', d',

d)

we are done.
Otherwise, d = d' = 0 and 12,13 are constant multiples of Ii
Then the problem is again trivial.
and

Solution 99, Alternative 2


Define the vector
v(x) = (1i(x),12(x),13(x))
for x C R.
If the v(x) span a proper subspace of R3, we cart find a vector (ci, c2, c3)

orthogonal to that subspace, and then cifi(x) + c2f2(x) + c3f3(x) =

for all x R.
So suppose the v(x) span all of R3.
Then
exist x1 < X2 < X3 c R such that v(xi), v(x2), v(x3) are
has
linearly independent, and so the 3 x 3 matrix A with
=
linearly independent rows.
But then A is invertible, and its columns also span R3.
This means we can find
c2, c3 such that

= (0,1,0),
and the function cif1 +c2f2 +c3f3 is then not monotonic, a contradiction.

4. Solutions to Advanced Problems

127

Problem 100 [USAMO 1999 proposal, Richard Stong]


Let x1,x2,. .
be variables, and let Y1,Y2,.
be the sums of
nonempty subsets of
be the kth elementary symmetric polynomial in
Let pk(xl,.
the (the sum of every product of k distinct yj'S).
.

For which k and n is every coefficient of Pk (as a polynomial in x1,..


even?
For example, ifn = 2, then Y1,Y2,Y3 are
+x2 and

Pi =Y1+Y2+Y32X1+2X2,
Y1Y2+Y2Y3+Y3Y1

P2
P3

Y1Y2Y3

Solution 100
We say a polynomial pk is even if every coefficient of Pk is even.
Otherwise, we say Pk is not even.

For any fixed positive integer n, we say a nonnegative integer k is bad


for n if k = for some nonnegative integer j.
We will show by induction on n that pk(xl, x2,
is not even if and
only if k is bad for ii.
,

For n =

1,

x1

k=1=212=r2.

is not even and k =

is bad for n =

Suppose that the claim is true for a certain n.

We now consider pk(x1,x2,

Let

0k(yl, Y2,.

.. Ys) be

the

elementary symmetric polynomial.

We have the following useful, but easy to prove, facts:


1.
2.

For all 1 r
,y8)

3.

= i+j=k
>

,Yr)Uj(Yr+1,

0k(X+Y1,X+y2,...,X+Y&)
=
iI<22<"<ik

{si

Sl<82<<S,.

,ik}

as

4. Solutions to Advanced Problems

128

=
Hence

=
i+j=k

+X2 +

. ,

r.O

By the induction hypothesis, every term of Pr(Xi, x2

xi,) is even un-

For such r, note that


2t

j - r) -

isevenunlessjr=Oorj--r=2t.
Therefore, taking coefficients modulo 2,

,Xn)Pj(X1,X2,'

i+j=k
,Xn)p2n_2t(x1,X2,

By the induction hypothesis, the terms in the first sum are even unless
In the second sum, every term appears twice except the term
Pk/2(X1,

for k even.

By the induction hypothesis, this term is even unless k/2 =


2v+1
some 0 <v n, that is k =
It follows that Pk(X1, x2,

is even unless k =

2w

for

for

some

4. Solutions to Advanced Problems

129

Furthermore, note that the odd coefficients in


Pk(X1,X2,
occur for different powers of

Therefore, the condition that k is bad for n + 1 is also sufficient for

to be odd.
Our induction is complete.

Problem 101 [Russia 20001


Prove that there exist 10 distinct real numbers a1, a2,..., a10 such that
the equation

(x ai)(xa2)...(x a10) =(x-i--ai)(x-i-a2).(x+aio)


has exactly 5 different real roots.

Solution 101
We show that
6,. . 2} is a group of numbers
a2,. .. a10} =
satisfying the conditions given in the problem.
The given equality becomes
,

(x 2)(x

1)x(x + 1)(x + 2)g(x2)

0,

where

g(u)

2[((7+6++3)u2+

(7.65+7.6.4+...--5.4.3)u+763].
no real solutions, then g(x2) = 0 has no real solutions.
If u1 and u2 are real solutions of g(u) = 0, then u1 -F-u2 <0 and U1U2 > 0,
that is, both u1 and u2 are negative.
It follows again that g(x2) = 0 has no real solutions.
Our proof is complete.

If g(u) =

has

GLOSSARY
Arithmetic-Geometric Mean Inequality (AMGM Inequality)
If a1, a2,..

n nonnegative

are

numbers, then

with

equality if

and

only if

al = a2 =

Binomial Coefficient
in the expansion of (x +

The coefficient of

is

(n'\
= k!(nk)!

Cauchy-Schwarz Inequality
For any

with

real

numbers

equality if

a1, a2,.. .

and only if

and b1, b2,..

and

are proportional,

= 1,2,.

..

, n.

Chebyshev Inequality
1. Let Xi, X2

and yi,

bers, such that x1 X2 <

be two sequences of real num-

and Yi y2

...

yn.

Then
+X2 +
2.

+Xn)(Yi +Y2+" +Yn)

+X2y2

Let Xl,X2...,Xn and Y1,Y2,..,Yn be two sequences of real numbers, such that x1 x2
..
xi-, and Yt y2
Then

+yn) Xlyl+X2y2+

+XnYn.

Glossary

132

De Moivre's Formula
For any angle a and for any integer n,

(cosa+
Elementary

= cosna+isinna.

Symmetric Polynomials (Functions)

Given indeterminates x1, ..., Xn, the elementary symmetric functions


are defined by the relation (in another indeterminate t)
Si,. ..
(t + x1) .. (t +

= tTh +

+ Sn.

taken k at a time. It
That is, Sk is the sum of the products of the
can be
is a basic result that every symmetric polynomial in x1
(uniquely) expressed as a polynomial in the

and vice versa.

Fibonacci Numbers
Sequence defined recursively by F1 = F2 = 1,

for

all

n E N.

Jensen's Inequality
1ff is concave up on an interval [a, b] and A1, A2, ...,
numbers with sum equal to 1, then

Aif(xi)+A2f(x2)+-..

are nonnegative

+).2X2+

in the interval [a. b]. If the function is concave


X2
down, the inequality is reversed.
for any x1 ,

Lagrange's Interpolation Formula


be ar. ,x7, be distinct real numbers, and let YO.Y1,. ..
bitrary real numbers. Then there exists a unique polynomial P(x) of
degree at most n such that
i = 0. 1,.. . , n. This is the
=
Let Xo,Xi,.

polynomial given by

i=O

x0)

Law of Cosines
Let ABC be a triangle. Then
BC2 = AB2 + AC2

- 2AB

ACcosA.

Glossary

133

Lucas' Theorem

Let p be a prime; let a and & be two positive integers such that

+ ak_lpkI +

a =

+ a0, b = bkpk +

b2 <p are integers for i =

where 0

(a'\

(ak\ (ak_1\

0,

bip + b0,

k. Then

(aj\ (ao\ (modp).

Pigeonhole Principle
If n objects are distributed among k < n boxes, some box contains at
least two objects.

Root Mean Square-Arithmetic Mean Inequality (RMSAM Inequality)


For positive numbers x1, X2,.

\/X?+X2++XkXl+X2++Xk
be any positive numbers for which ai +
we define
For positive numbers x1, x2,...

More generally, let a1, a2,.. .

a2 +

1.

= min{x1,x2,. .. ,xk},
Moc, =max{xl,x2,...,xk},

aj a2
0xl
x2
=
+
a non-zero real number. Then

<M8
for s

t.

M00

Glossary

134

Triangle Inequality

Let z =

+ bi be a complex number. Define the absolute value of z to

be

zI=
Let

is

and /3 be two complex numbers. The inequality

called the triangle inequality.

Let =
Then

and i9 = /9i

+/32i,

/92 are real numbers.

where

v = [/91,/92], and w =
Vectors u =
+192] form a
triangle with sides lengths
and + /31
The triangle inequality restates the fact that the length of any side of a
triangle is less than the sum of the lengths of the other two sides.

Vieta's Theorem
Let x1, X2,.

. ,

be the roots of polynomial

P(x) =
where
of

the

+... +

E C. Let 8k
#0 and a0,a1,
taken k at a time. Then
.

be

+ a0.

the sum of the products

=
that is,

x1x2

x1x2

a
sin a
cos a
1

Glossary

135

addition and subtraction formulas:

sin(ab) =sinacosbcosasinb,
cos(ab)
a tan b
tan(ab)= tan tan
a tan b'
1

double-angle formulas:

sin2a = 2sinacosa,
cos 2a =

tan2a=

cos2 a sin2

a = 2 cos2 a

2 tan a

1tan2a'

triple-angle formulas:

sin3a = 3sina 4s1n3 a,


cos3a = 4cos3a 3cosa,
tan3
tan3a= 3tana 2 a

13tan a

half-angle formulas:
2 tan

sina=
1

tan2

tan2

a+b

cosa+cosb=
tana +tanb =

ab

sin(a+b)
cosacosb'

difference-to-product formulas:
sin a sin b = 2 sin

ab

cos

a+b

2 sin2

a,

Glossary

136

cos a cos b 2 sin

tanatanb=

ab

sin

a+b

sin(a b)
cos a cos b

product-to-sum formulas:

2sinacosb = sin(a + b) +sin(a b),


2cosacosb = cos(a + b) cos(a
2sinusinb = cos(a + b) + cos(a b).

FURTHER READING

1.

Andreescu, T. Kedlaya, K.; Zeitz, P., Mathematical Contests


1995-1996: Olympiad Problems from around the World, with
Solutions, American Mathematics Competitions, 1997.

2. Andreescu, T. Kedlaya, K., Mathematical Contests 1996-1997:

Olympiad Problems from around the World, with Solutions,


American Mathematics Competitions, 1998.
3. Andreescu, T. Kedlaya, K., Mathematical Contests 1997-1998:

Olympiad Problems from around the World, with Solutions,


American Mathematics Competitions, 1999.

4. Andreescu, T. Feng, Z., Mathematical Olympiads: Problems and


Solutions from around the World, 1998-1999, Mathematical
Association of America, 2000.
5. Andreescu, T. Gelca, R., Mathematical Olympiad Challenges,
Birkhuser, 2000.
6. Barbeau, E., Polynomials, Springer-Verlag, 1989.

7. Beckenbach, E. F., Bellman, R., An Introduction to Inequalities,


New Mathematical Library, Vol. 3, Mathematical Association of
America, 1961.

8. Chinn, W. C., Steenrod, N. E., First Concepts of Topology, New


Mathematical Library, Vol. 27, Random House, 1966.
9. Cofman, J., What to Solve?, Oxford Science Publications, 1990.

10. Coxeter, H. S. M., Greitzer, S. L., Geometry Revisited, New

Mathematical Library, Vol. 19, Mathematical Association of


America, 1967.
11. Doob, M., The Canadian Mathematical Olympiad 1969-1993,
University of Toronto Press, 1993.
12. Engel, A., Problem-Solving Strategies, Problem Books in
Mathematics, Springer, 1998.
13. Fomin, D., Kirichenko. A., Leningrad Mathematical Olympiads
1987-1991, MathPro Press, 1994.

Further Reading

138

14. Fomin, D., Genkin, S., Itenberg, 1., Mathematical Circles,


American Mathematical Society, 1996.

15. Graham, R. L., Knuth, D. E., Patashnik, 0., Concrete


Mathematics, Addison-Wesley, 1989.

16. Greitzer, S. L., International Mathematical Olympiads, 1959-1977,


New Mathematical Library, Vol. 27, Mathematical Association of
America, 1978.
17. Grossman, I., Magnus, W., Groups and Their Graphs, New

Mathematical Library, Vol. 14, Mathematical Association of


America, 1964.

18. Kazarinoff, N. D., Geometric Inequalities, New Mathematical


Library, Vol. 4, Random House, 1961.
19. Klamkin, M., International Mathematical Olympiads, 1978-1985,
New Mathematical Library, Vol. 31, Mathematical Association of
America, 1986.

20. Klamkin, M., USA Mathematical Olympiads, 1972-1986, New


Mathematical Library, Vol. 33, Mathematical Association of
America, 1988.
21. Kiirschk, J., Hungarian Problem Book, Volumes I & II, New
Mathematical Library, Vols. 11 & 12, Mathematical Association of
America, 1967.

22. Kuczma, M., 144 Problems of the Austrian-Polish Mathematics


Competition 1978-1998, The Academic Distribution Center, 1994.
23. Larson, L. C., Problem-Solving Through Problems, Springer-Verlag,
1983.

24. Lausch, H., Bosch Giral, C., The Asian Pacific Mathematics Olympiad 1989-2000, AMT Publishing, Canberra, 2001.
25. Liu, A., Chinese Mathematics Competitions and Olympiads
1981-1993, AMT Publishing, Canberra, 1998.

26. Lozansky, E., Rousseau, C. Winning Solutions, Springer, 1996.

27. Ore, 0., Graphs and their uses, Random House, 1963.
28. Ore, 0., Invitation to number theory, Random House, 1967.
29. Sharygin, I. F., Problems in Plane Geometry, Mir, Moscow, 1988.

Further Reading

139

30. Sharygin, I. F., Problems in Solid Geometry, Mir, Moscow, 1986.

31. Shklarsky, D. 0, Chentzov, N. N; Yaglom, I. M., The USSR


Olympiad Problem Book, Freeman, 1962.

32. Slinko, A., USSR Mathematical Olympiads 1989-1992, AMT


Publishing, Canberra, 1997.

33. Soifer, A., Colorado Mathematical Olympiad: The first ten years,
Center for excellence in mathematics education, 1994.
34. Szekely, C. J., Contests in Higher Mathematics, Springer- Verlag,
1996.

35. Stanley, R. P., Enumerative Combinatorics, Cambridge University


Press, 1997.

36. Taylor, P. J., Tournament of Towns 1980-1984, AMT Publishing,


Canberra, 1993.
37. Taylor, P. J., Tournament of Towns 1984-1 989, AMT Publishing,
Canberra, 1992.

38. Taylor, P. J., Tournament of Towns 1989-1993, AMT Publishing,


Canberra, 1994.
39. Taylor, P. .1., Storozhev, A., Tournament of Towns 1993-1997,
AMT Publishing, Canberra, 1998.
40. Tomescu, I., Problems in Combinatorics and Graph Theory, Wiley
1985.

41. Vanden Eynden, C., Elementary Number Theory, McGraw-Hill,


1987.

42. Wilf, H. S., Generatingfunctionology, Academic Press, 1994.

43. Wilson, R., Introduction to graph theory, Academic Press, 1972.

44. Yaglom, 1. M., Geometric Transformations, New Mathematical


Library, Vol. 8, Random House, 1962.
45. Yaglom, I. M., Geometric Transformations II, New Mathematic
Library, Vol. 21, Random House, 1968.
46. Yaglom , I. M., Geometric Transformations III, New Mathemati
Library, Vol. 24, Random House, 1973.

47. Zeitz, P., The Art and Craft of Problem Solving, John Wiley &
Sons, 1999.

Titu Andreescu is
Director of the

A met jean

tics
Competitions,
serves

as

Head

Coach of the USA

international
Mathem atical
Olympiad

(lMO)

Team, is Chair of the USA Mathematical


Olympiad Committee, and is Director of the
USA Mathematical Olympiad Summer

Program. Originally from Romania, Prof.


Andreescu received the Distinguished
Professor Award from the Romanian

Ministty of Education in 1983, then after


moving to the USA was awarded the Edyth

May Sliffe Award for Distinguished High


School Mathematics Teaching from the
MM in 1994. in addition, he received a
Certificate of Appreciation presented by the

President of the MM for This outstanding


service as coach of the USA Mathematical
Olympiad Program in prepanng the USA
team for its perfect performance in Hong
Xong at the 1994 1MO".

Zuming

Feng

graduated with a
PhD from Johns
Hopkins University

with an
on
Algebraic
Nuniber Theory
and
Elliptic
Curves. He teaches

at Phillips Exeter
Academy. He also serves as a coach of the
USA international Mathematical Olympiad
(IMO) Team, is a member of the USA
Mathematical Olympiad Committee, and is
an
assistant director of the USA
Mathematical Olympiad Summer Program.
He received the Edyth May Sliffe Award for
THF AUSTRALIAN MATHEMATiCS TRUST

Distinguished High School Mathematics


Teaching from the MM in 1996.

ENRICHMENT SERIES

1S8N187642012X

You might also like